Behavioral Health - ROSH & SmartyPance EOR Practice Q's

Ace your homework & exams now with Quizwiz!

A 26-year-old man is sexually aroused by exposing his genitals to nearby drivers while in traffic. Which of the following paraphilic disorders is the most likely diagnosis? A. Exhibitionistic disorder B. Frotteuristic disorder C. Sexual masochism D. Voyeuristic disorder

A. Exhibitionistic disorder Frotteuristic disorder (B) is a paraphilic disorder defined by recurrent and intense sexual arousal from touching or rubbing one's genitals against a nonconsenting person. Sexual masochism (C) is a paraphilic disorder characterized by sexual arousal from being humiliated. Voyeuristic disorder (D) is a paraphilic disorder characterized by recurrent and intense sexual arousal from observing an unsuspecting person who is naked, disrobing, or involved in sexual activity. Exhibitionistic disorder is a paraphilic disorder characterized by an individual becoming aroused from exposing their genitals to an unsuspecting person. Individuals with exhibitionistic disorder frequently masturbate during or after the exposure. The diagnosis requires either the person to have acted on these urges within the past 6 months with a nonconsenting person or to experience significant distress or impairment. Therefore, individuals who have exhibitionistic urges or desires but do not act on them are not diagnosed with exhibitionistic disorder unless there is significant distress. Exhibitionistic disorder most often begins prior to 18 years of age. It is more prevalent among males, which is consistent with other paraphilic disorders. Treatment options include pharmacotherapy and psychotherapy. Selective serotonin reuptake inhibitors are helpful in some cases, while long-acting agonists of luteinizing hormone and antiandrogens may be used in severe cases. Cognitive behavioral therapy is the recommended psychotherapy. Patients with exhibitionistic disorder who receive treatment frequently improve. However, the course is typically chronic when untreated.

A 22-year-old man presents to the emergency department in a catatonic state. He was found at home with a few white tablets in his hand. Vital signs show tachypnea, tachycardia, hyperpyrexia, and hypertension. Physical exam shows vertical nystagmus and hyperreflexia. Laboratory results show elevated creatine kinase. Which of the following substances did this patient most likely ingest? A. 3-Methoxyphencyclidine B. Heroin C. Lysergic acid diethylamide D. Tetrahydrocannabinol

A. 3-Methoxyphencyclidine Phencyclidine is a dissociative anesthetic that has been used as a recreational drug since the 1970s. In recent years, synthetic derivatives of phencyclidine have been produced, such as 3-Methoxyphencyclidine (3-MeO-phencyclidine) and 4-Methoxyphencyclidine (4-MeO-phencyclidine), that offer more potent dissociative effects. Phencyclidine and its derivatives are antagonists of the N-methyl-D-aspartate receptors in the brain and also inhibit the reuptake of serotonin, norepinephrine, and dopamine. Phencyclidine and its derivatives also bind to the sigma receptor complex. Phencyclidine is sold in tablet, powder, crystal, or liquid form and can be smoked, insufflated, ingested, or injected. 3-MeO-phencyclidine is sold in tablet form. Signs and symptoms of intoxication with phencyclidine and its synthetic derivatives can include hallucinations, psychosis, catatonia, coma, agitation, hyperreflexia, hyperpyrexia, tachycardia, elevated blood pressure, disorientation, seizures, and nystagmus (horizontal, vertical, or rotatory). Common laboratory abnormalities associated with phencyclidine intoxication include elevated creatine kinase (indicative of rhabdomyolysis), hypoglycemia, elevated liver transaminases, and hyperuricemia. A urine drug test will detect PCP that has been used within about the last 14 days. Treatment for acute intoxication with phencyclidine and its derivatives is supportive care, with airway protection and benzodiazepines for agitation or seizures. Heroin (B) is an opioid that is either snorted, smoked, or injected. A urine toxicology screen will be positive for opioids in a patient who is acutely intoxicated with heroin. Lysergic acid diethylamide (C) is a hallucinogenic drug that may cause altered mental status but will not usually cause great alterations in vital signs. Lysergic acid diethylamide is not included in a general urine toxicology screen. Tetrahydrocannabinol (D) is the chemical name for marijuana. Marijuana is a cannabinoid that binds to CB1 receptors in the brain, causing euphoria, relaxation, increased hunger, and heightened sensory perception. Lactic acidosis, hyperreflexia, and hyperpyrexia are not common in tetrahydrocannabinol intoxications. Tetrahydrocannabinol will appear on a urine toxicology screen.

A 4-year-old boy presents to the clinic for his annual physical. His vital signs are within normal limits, and the physical exam is normal. He avoids eye contact with the clinician and with his mother, speaks only in one- or two-word sentences, and is preoccupied with a toy car he has brought to the office. His mother states he has never liked to be hugged or touched, and he prefers to play by himself. Which of the following is a risk factor for the most likely diagnosis? A. Advanced parental age B. Infants born after 42 weeks gestation C. Measles, mumps, and rubella vaccination D. Slow rate of head growth during infancy

A. Advanced parental age Autism spectrum disorder refers to a neurodevelopmental disorder characterized by impaired social interaction and repetitive patterns of behavior. Risk factors for the development of autism spectrum disorder include male sex, advanced parental age (maternal or paternal), family members with autism, certain genetic syndromes, and brain abnormalities. Diagnosis of autism spectrum disorder can be made when a patient has deficits in social communication and interaction involving emotional reciprocity, nonverbal communication, and maintaining relationships. The patient must also show restricted or repetitive patterns of behavior in two or more areas, including repetitive movements (such as rocking or flapping), repetitive speech patterns, ritualized behavior, highly fixated interests, and indifference to or hyperresponsiveness to environmental aspects like sound or temperature. Treatment of autism spectrum disorder includes behavioral and educational interventions that aim to improve social function, develop independence, decrease negative behaviors, and improve academic functioning. Certain pharmacologic agents may be prescribed to limit self-injurious behaviors or diminish hyperactivity. These include stimulants, such as methylphenidate, alpha-2 adrenergic agents, such as guanfacine, and antipsychotics, such as risperidone. Patients with autism spectrum disorder may show a range of symptom severity, with some patients able to function independently and others needing lifelong assistance.

A 22-year-old woman with a history of congenital prolonged QT interval presents to the clinic with dysphoria for the past 2 months. She reports she has felt fatigued, been sleeping more than usual, and had decreased appetite. She has not been interested in doing activities she previously viewed as hobbies. Which of the following medications to treat her suspected condition would require an electrocardiogram prior to initiation? A. Amitriptyline B. Duloxetine C. Escitalopram D. Venlafaxine

A. Amitriptyline Duloxetine (B) and venlafaxine (D) are serotonin and norepinephrine reuptake inhibitors. They do not cause prolongation of the QT interval. The common side effects of serotonin and norepinephrine reuptake inhibitors include nausea, constipation, dizziness, dry mouth, diaphoresis, and sexual dysfunction. There is also a small increased risk of bleeding and bone resorption, particularly in patients on blood thinners or who have osteoporosis, respectively. Escitalopram (C) is a selective serotonin reuptake inhibitor. Selective serotonin reuptake inhibitors may infrequently prolong the QT interval but do not require an electrocardiogram prior to use. The common adverse effects of selective serotonin reuptake inhibitors are sexual dysfunction, drowsiness, insomnia, weight gain, headache, dizziness, anxiety, and dry mouth. Risk of abnormal heart rhythms, such as heart block and ventricular dysrhythmias, is one of the safety concerns associated with tricyclic antidepressants because these medications can prolong the QT interval on electrocardiogram. Therefore, patients should undergo a cardiac evaluation prior to the prescription of a tricyclic antidepressant. The cardiac evaluation should include a thorough history in all patients and a baseline ECG in patients with concerns identified on history and in all patients who are at least 40 years of age. The history should include screening the patient if they have any cardiac symptoms (such as chest pain, palpitations, shortness of breath, dyspnea on exertion, and syncope), known cardiac disease (e.g., congenital or acquired prolonged QT interval, coronary artery disease), or family history of heart disease. It is important to know which additional medications a patient is taking to see whether they are taking other medications that are known to prolong the QT interval. The patient in the vignette requires an ECG because she has a history of a congenital prolonged QT interval. Tricyclic antidepressants are frequently associated with other side effects, such as anticholinergic effects (blurred vision, constipation, dry mouth, and urinary retention), antihistaminic effects (increased appetite leading to weight gain, sedation, confusion, and delirium), decreased seizure threshold, sexual dysfunction, diaphoresis, and tremor. These adverse effects are caused by the interactions that tricyclic antidepressants have with a variety of neurotransmitter systems, such as muscarinic, histamine, and alpha-adrenergic receptors, in addition to the serotonin and norepinephrine receptors, which give tricyclic antidepressants their efficacy. Ask patients about physical symptoms prior to starting an antidepressant since depression itself can cause physical symptoms. An additional safety concern with tricyclic antidepressants is that they are more dangerous in overdose than other classes of antidepressants due to the prolongation of the QT interval. This is particularly concerning in patients with depression since they are at higher risk for suicide.

A 27-year-old man presents to the emergency department reporting chest pain. On physical exam, the patient is agitated and diaphoretic with dilated pupils. Current vital signs are temperature 101.2°F, BP 160/95 mm Hg, HR 124 beats per minute, RR 24 breaths per minute, and oxygen saturation 94% on room air. Which of the following is the most likely diagnosis? A. Amphetamine intoxication B. Anticholinergic intoxication C. Opioid intoxication D. Phencyclidine intoxication

A. Amphetamine intoxication Amphetamine intoxication is caused by the ingestion of phenethylamine, including traditional amphetamines and newer synthetic compounds. Amphetamines are lipophilic compounds that readily cross the blood-brain barrier and result in the stimulation of alpha- and beta-adrenergic receptors. Phenylethylamines cause the release of dopamine, serotonin, and norepinephrine and also inhibit reuptake. The excess dopamine and serotonin may cause psychotic symptoms. Ring substitutions and other designer modifications have been produced to cause various psychoactive properties. Intoxication with amphetamines or cathinone results in hyper-alertness, hypertension, tachycardia, mydriasis, diaphoresis, hyperthermia, and alterations in mental status, including anxiety, agitation, violent behaviors, and seizures. In high doses, hypervigilance, paranoid ideation, bruxism, tactile hallucinations of insect infestations, persecutory delusions, and aggressive responses may occur. In addition, overdose with amphetamines can produce persistent myoclonus and tremors that may result in rhabdomyolysis. Synthetic cathinones may result in acute kidney injury with acute tubular necrosis and kidney vasospasm resulting in kidney ischemia. Electrolyte abnormalities may also be seen, including hypokalemia, hyponatremia, hypermagnesemia, and elevated anion gap acidosis. Patients who inject amphetamines may have associated cellulitis, abscess formation, and necrotizing fasciitis. While specific testing is helpful to confirm the suspicion of intoxication with a specific substance, amphetamine and synthetic cathinone toxicity is often a clinical diagnosis, as serum drug concentrations are not always available in a timely manner. With an accurate history of amphetamine ingestion and present symptoms consistent with intoxication, a diagnosis can be made. Other recommended general testing includes fingerstick glucose, acetaminophen and salicylate levels, electrocardiogram, pregnancy test in women of childbearing age, serum electrolytes, creatine kinase and urine myoglobin, serum creatinine, serum aminotransferase concentrations, and coagulation studies. Treatment consists of protecting the patients airway and control of agitation and sympathetic excess. Hydration and assessment of nutritional status are also important. Gastrointestinal decontamination may be useful in cases of oral ingestion of large doses of amphetamines but otherwise provides minimal benefit. First-line therapy for agitation is IV benzodiazepines, which also treat the hypertension and seizures often seen in these patients. Benzodiazepines can also lessen the excessive muscle activity that generates heat contributing to hyperthermia. In some cases of hyperthermia, neuromuscular paralysis and evaporative cooling or ice packs may also be necessary. Anticholinergic intoxication (B) presents similarly to amphetamine intoxication with hyperthermia, tachycardia, dilated pupils and flushed skin. However, these patients do not have diaphoresis but rather have dry skin. Opioid intoxication (C) often presents with decreased consciousness, pinpoint pupils, and respiratory depression. Phencyclidine intoxication (D) may cause agitation and psychosis and is not generally accompanied by the adrenergic symptoms seen with amphetamine intoxication, such as hypertension, tachycardia, hyperthermia and diaphoresis.

A 19-year-old man presents to the emergency department in police custody with dog bites on his left arm. The police were called because the patient was attempting to stab pets in the neighborhood. The police indicate that the patient has a history of vandalism and setting public buildings on fire. You examine the patient, and he demonstrates no remorse for his actions. Which of the following is the most likely diagnosis? A. Antisocial personality disorder B. Avoidant personality disorder C. Borderline personality disorder D. Narcissistic personality disorder

A. Antisocial personality disorder Avoidant personality disorder (B) is a cluster C personality disorder that is marked by social inhibition, feelings of inadequacy, and hypersensitivity to criticism. These individuals often have few social relationships despite a desire for social interaction. Borderline personality disorder (C) is marked by impulsive behavior, unstable mood and interpersonal relationships, fear of abandonment, and increased risk of suicidal behavior. Narcissistic personality disorder (D) is marked by grandiosity, excessive need for admiration, lack of empathy, and exploitative relationships. Antisocial personality disorder is a cluster B personality disorder along with borderline, histrionic, and narcissistic personality disorders. Cluster B personality disorders tend to be marked by dramatic, emotional, or erratic behavior. Antisocial personality disorder presents with a pervasive pattern of exploiting others, rule-breaking, violating the law, and deceitfulness. Individuals with antisocial personality disorder lack empathy or remorse for their actions. Patients usually develop symptoms initially during childhood, although antisocial personality disorder cannot be officially diagnosed until 18 years of age. However, symptoms often seen during childhood include fighting with peers, conflict with parents, stealing, vandalism, fire setting, cruelty to animals or other children, poor academic performance, and running away from home. Adults diagnosed with antisocial personality disorder frequently achieve poor job performance, pathologically lie, display sexually promiscuous behavior, and have unstable marriages. Antisocial personality disorder is diagnosed according to the criteria in the fifth edition of the Diagnostic and Statistical Manual of Mental Disorders (DSM-5). The treatment for antisocial personality disorder varies some based on the severity of symptoms and the patient's insight and motivation for improvement. Although cognitive behavioral therapy has not demonstrated efficacy in clinical studies, patients who have mild symptoms and a desire to be treated may benefit from therapy. Pharmacologic therapy is generally reserved for treating psychiatric comorbidities and patients with symptoms of severe aggression. Second-generation antipsychotics, such as risperidone and quetiapine, are the preferred agents for patients with severe aggression.

A 23-year-old woman with schizophrenia presents to the clinic for follow-up after beginning risperidone 5 months ago. The patient reports decreased symptoms after starting risperidone, but she has not menstruated for 3 months and has noticed an increased frequency of headaches. Which of the following medications is most appropriate for continued treatment? A. Aripiprazole B. Clozapine C. Paliperidone D. Pimozide

A. Aripiprazole Schizophrenia is a psychiatric disorder with associated social and occupational functioning impairments secondary to psychotic symptoms. Positive symptoms of schizophrenia include hallucinations and delusions, while negative symptoms present as flattened affect, alogia, apathy, and anhedonia. Patients with schizophrenia often display disorganized speech, including tangential speech, circumstantial speech, derailment, neologisms, and word salad. According to the fifth edition of the Diagnostic and Statistical Manual of Mental Disorders, patients must present with at least 1 month of delusions, hallucinations, or disorganized speech in addition to either negative symptoms or grossly disorganized or catatonic behavior. Functional capacity must be impaired by these symptoms for a period of at least 6 months (wherein at least 1 month consists of the active-phase symptoms described above). Clinicians should rule out mood disorders with psychotic features. Specifiers can be added to clarify how many episodes the patient has experienced (i.e., first episode or multiple episodes) and whether the condition is in the acute phase or in partial or full remission. First-line treatment for schizophrenia involves antipsychotic medications, with second-generation agents (e.g., aripiprazole, paliperidone, quetiapine, risperidone) generally preferred due to fewer extrapyramidal side effects than first-generation agents (e.g., chlorpromazine, haloperidol, pimozide). The most common extrapyramidal side effects include pseudoparkinsonism, acute dystonic reactions, akathisia, and tardive dyskinesia. Risperidone, a frequently administered second-generation antipsychotic, may also cause hyperprolactinemia leading to hypogonadism (e.g., oligomenorrhea, amenorrhea, galactorrhea, decreased bone mineral density). Patients with these manifestations should be switched to alternative antipsychotic medications with a lower probability of prolactin elevation, such as aripiprazole. Treatment-resistant schizophrenia is often treated with clozapine, but this agent can cause fatal agranulocytosis and should be reserved for those who cannot tolerate other agents or who are refractory to multiple medical approaches to therapy. Clozapine (B) is reserved for use in patients with residual symptoms despite medical therapy and is not appropriate for the patient in the vignette. Paliperidone (C) is a second-generation antipsychotic that has a high potential of causing hyperprolactinemia and thus would not benefit the patient in the vignette, who is experiencing these related side effects. Similarly, pimozide (D), a first-generation antipsychotic, has a higher probability of causing elevated prolactin levels.

A 5-year-old boy presents to the clinic with his father who reports persistent behavioral issues since age 4 that have been recently worsening. The patient's father is concerned about daily temper tantrums and disobedient behavior at home, including arguing with his parents and deliberately annoying his siblings. He also reports teacher concerns about irritable mood and disregard for classroom rules on an almost daily basis. Which of the following interventions is most appropriate for this patient given the most likely diagnosis? A. Assess psychosocial situation and parent training B. Begin cognitive behavioral therapy C. Begin pharmacological treatment with stimulant medication D. Refer for inpatient evaluation and treatment

A. Assess psychosocial situation and parent training While beginning cognitive behavioral therapy (B) is an important aspect of treatment of ODD, evaluating the patient's psychosocial situation and training parents is priority to identify potential causes and triggers of the behavior. Beginning pharmacological treatment with stimulant medication (C) is only appropriate for patients who also meet diagnostic criteria for ADHD. Referring for inpatient evaluation and treatment (D) is not indicated for any patient who is stable and not a threat to themselves or others. Oppositional defiant disorder (ODD) is a conduct disorder in childhood involving angry mood, argumentative or defiant behavior, and vindictiveness. Genetic predisposition is not certain, but the disorder is more common in boys than girls. It is thought that a child's upbringing and social environment play a role. For example, harsh, inconsistent, or neglectful child-rearing practices are common in families of children and adolescents with ODD. Temperamental factors related to problems in emotional regulation, such as high levels of emotional reactivity and poor frustration tolerance, have also been associated with ODD. ODD is among the most common reasons for referral to mental health professionals, affecting about 3% of all children. Over half of those who meet ODD criteria will no longer do so several years later. However, conduct disorder will develop in about one-third of these patients, especially in those with early-onset, severe symptoms, or comorbid attention-deficit/hyperactivity disorder (ADHD). Patients with ODD are typically angry and irritable, often disobeying authority figures, arguing with them, and refusing to comply with rules, often to annoy. For diagnosis, a patient must demonstrate at least four symptoms from the following categories: angry or irritable mood, defiant or argumentative behavior, or vindictiveness lasting at least 6 months. Symptoms must cause functional impairment to the patient, typically interfering with education and social functioning, and other differential diagnoses must be ruled out before a diagnosis of ODD is appropriate (ADHD, substance use, psychotic or mood disorders, ordinary childhood growth and development). Severity depends on the number of settings that symptoms occur in: symptoms of mild cases occur in only one location (i.e., home, school, or with friends), moderate cases occur in two or more locations, and severe cases occur in three or more locations. Treatment consists of parent training combined with outpatient psychological therapy. It is important to evaluate the child's psychosocial environment and also provide proper support to parents, as they may experience adverse mental health effects due to the difficult behavior and social disruption. Parent training is the most important aspect of treatment as it teaches parents to be more positive and less harsh in their discipline style. Collaborative problem-solving interventions facilitate joint problem solving rather than teaching children to comply with parental demands. Some studies have found that stimulant medications used to treat attention-deficit/hyperactivity disorder may be helpful in cases of oppositional defiant disorder with concomitant ADHD. However, studies have not shown that stimulants reduce the symptoms of ODD when ADHD is absent.

A 4-year-old boy presents to the clinic with his mother, who has noticed he has more difficulty sitting still than her other children did at a similar age. His preschool teacher says he frequently interrupts other children and struggles to remain seated after being told to do so. Which of the following is the most likely suspected diagnosis? A. Attention-deficit/hyperactivity disorder hyperactive subtype B. Attention-deficit/hyperactivity disorder inattentive subtype C. Conduct disorder D. Oppositional defiant disorder

A. Attention-deficit/hyperactivity disorder hyperactive subtype Behavioral therapy is the first-line treatment for preschool-aged children, while stimulants, such as methylphenidate, are the first-line treatment for school-aged children or adolescents. ADHD inattentive subtype (B) is typically not diagnosed until children are closer to 8 years of age. The symptoms of the patient in the vignette are more consistent with the hyperactive subtype than the inattentive subtype. Conduct disorder (C) is a severe childhood behavioral condition that is marked by destruction of property, aggression toward people or animals, deceitfulness or theft, and serious violation of rules. Oppositional defiant disorder (D) is a maladaptive pattern of defiance or anger seen in children that results in distress for the patient or caregiver. The manifestations of oppositional defiant disorder are less severe than conduct disorder and do not involve the destruction of property, violation of the law, or physical violence against others.

A 3-year old boy presents to the pediatrician with his mother. She reports the preschool he goes to is concerned about his behavior. She notes the patient has difficulty remaining seated during lessons, is always "on the go," blurts out the answers to questions without raising his hand, talks to his classmates during lesson times even after correction, and cannot wait his turn when required. Which of the following is the recommended first-line treatment for this patient's suspected diagnosis? A. Behavioral interventions B. Elimination diet C. Pharmacotherapy D. Psychotherapy

A. Behavioral interventions The diagnosis of attention-deficit/hyperactivity disorder (ADHD) can be determined when a patient < 17 years of age has more than six symptoms of hyperactivity and impulsivity. These symptoms must occur often for at least 6 months and be present in more than one setting. The patient must have had symptoms before 12 years old, and the symptoms should impair function in academic, occupational, or social activities. For preschool-aged children, behavioral therapy is the recommended initial intervention. This can include positive reinforcement and withdrawing privileges due to unwanted behavior. Parent-child behavioral therapy may also be beneficial. Many patients with ADHD do better with a clear daily schedule and consistent small target goals. Pharmacotherapy can be used as an adjunct therapy in preschool-aged children and is the first-line treatment for children ≥ 6 years of age. However, behavioral interventions are also used in school-aged children, with the medication to better reduce symptoms. Elimination diets (B) can be considered on a case-by-case basis, but the influence on the symptoms of ADHD is controversial. It is not the recommended first-line therapy. It can be considered if caregivers are concerned about medication side effects or use and are motivated to adhere to the diet. Pharmacotherapy (C) is the first-line treatment for school-aged children. Psychotherapy (D) is not generally a recommended treatment for patients with ADHD unless there are coexisting conditions, such as anxiety, depression, or social deficits. It has not been shown to be beneficial for the core symptoms of ADHD.

A 23-year old woman presents to your office for a new-patient appointment and expresses repeated concern over the appearance of her nose. She mentions she has had two prior nose plastic surgeries by different surgeons but is still unsatisfied. Throughout the visit, she continually uses a pocket mirror to check her nose and also notes she was recently fired from her job due to decreased work productivity. What is the most likely diagnosis? A. Body dysmorphic disorder B. Factitious disorder C. Illness anxiety disorder D. Obsessive-compulsive disorder

A. Body dysmorphic disorder Factitious disorder (B) is a mental disorder characterized by a patient or another person presenting a patient with a falsified medical condition in an attempt to deceive a medical clinician. The falsified condition may involve injuring themselves or causing body changes in an attempt to deceive the clinician to gain attention, sympathy, or reassurance without external incentives. Illness anxiety disorder (C), previously hypochondriasis, is a condition in which the patient is preoccupied with having or acquiring a serious illness but does not have a preoccupation with body appearance. Obsessive-compulsive disorder (D) is a disorder characterized by intrusive and recurrent thoughts (obsessions) occurring for hours each day, as well as repetitive behaviors (compulsions), including hand-washing or counting, that attempt to suppress the obsessive thoughts. The obsessions are not around body appearance, unlike body dysmorphic disorder. Body dysmorphic disorder can be suspected when someone has a preoccupation with at least one defect of a physical attribute that may be nonexistent or slight. This preoccupation leads to repetitive behaviors, including repetitive mirror checking, excessive grooming and skin-picking, and camouflaging, which leads to significant psychological distress. The psychosocial distress causes the patient to possibly avoid work or social situations due to concerns of other people making fun of them due to their perceived body defect. These patients have a history of repeated cosmetic procedures or have seen several medical clinicians that were unsatisfactory to them. In addition, the anxiety or depression is resistant to medication therapy. Patients who are found to meet the criteria of body dysmorphic disorder have varying degrees of insight from good or fair insight, in which the patient is able to recognize the beliefs are not true, to delusional beliefs, in which the patient believes the perceived body defect is true. The face or individual parts of the face, such as the nose, are the most common features for patients to fixate on.

A 20-year-old woman presents to the clinic due to feeling cold and having irregular menses, stating that her last menstrual cycle was over 6 months ago. Her body mass index is 17 kg/m2, and her urine pregnancy test is negative. She states she exercises every day for at least 2 hours because she is overweight and is terrified of gaining weight. Which of the following physical exam findings is most consistent with the diagnosis? A. Bradycardia B. Eroded teeth enamel C. Hypertension D. Oily skin

A. Bradycardia Eroded teeth enamel (B) is more likely to be present in bulimia nervosa than anorexia nervosa. Hypotension, not hypertension (C), would normally be seen in anorexia nervosa. Oily skin (D) is not seen in anorexia nervosa, rather dry skin or xerosis is seen due to dehydration.

A 24-year-old woman presents to the clinic complaining of palpitations and depressed mood. On physical exam, she is visibly cachectic with a body mass index of 13 kg/m2. A systolic murmur and systolic click are appreciated at the cardiac apex, and her skin is cool and dry with an abundance of fine, dark hair over her entire body. The patient reports eating only one small salad per day because she has an intense fear of becoming fat. The patient is admitted for evaluation and treatment. Which of the following medications should be avoided? A. Bupropion B. Buspirone C. Olanzapine D. Sertraline

A. Bupropion Buspirone (B) is an anxiolytic that may be used to treat patients with anorexia nervosa who also have generalized anxiety disorder. Olanzapine (C) is an antipsychotic medication with a side effect of weight gain that is sometimes used in patients with anorexia nervosa who do not gain sufficient weight while undergoing refeeding. Sertraline (D) is a selective serotonin reuptake inhibitor that may be used to treat generalized anxiety or major depression in patients with anorexia nervosa. This class of medications, however, has not been shown to be effective in the treatment of body dysmorphic disorder or anorexia nervosa itself. Anorexia nervosa is an eating disorder characterized by severe calorie restriction, low body mass index, and obsessive thoughts about food and weight gain. Patients with anorexia nervosa have an intense fear of gaining weight accompanied by a distorted view of their own bodies. Comorbid psychiatric disorders are common in patients with anorexia nervosa and include anxiety, depression, body dysmorphic disorder, obsessive-compulsive disorder, and post-traumatic stress disorder. Female patients are more likely to be affected with anorexia nervosa than male patients, and the average age of onset is 18 years. Medical complications of anorexia nervosa can include cardiac muscle atrophy, mitral valve prolapse, pericardial effusion, bradycardia, osteoporosis, amenorrhea, gastroparesis, constipation, brain atrophy, pancytopenia, hyperpigmentation, xerosis, lanugo, petechiae, and yellowing of the skin. Patients with anorexia nervosa require hospitalization if vital signs are not stable; body mass index is under 14 kg/m2; evidence is found of cardiac, kidney, or hepatic compromise; or there is marked dehydration. Treatment for anorexia nervosa is with careful refeeding and psychotherapy. Pharmacotherapy is aimed at specific medical complications or psychological comorbidities. Patients with anorexia nervosa who suffer concomitant generalized anxiety, obsessive-compulsive disorder, or major depression should not be given the atypical antidepressant bupropion, as an increased incidence of seizure activity is found when bupropion is administered to patients with an eating disorder.

A 23-year-old woman gives birth to a child with spina bifida after being on an unknown medication during the first trimester of pregnancy. The medication was prescribed to treat a psychiatric condition marked by hypomanic and major depressive episodes. Which of the following medications is most likely to cause spina bifida in the fetus when used during pregnancy? A. Carbamazepine B. Lamotrigine C. Lithium D. Olanzapine

A. Carbamazepine Lamotrigine (B) is an antiseizure medication sometimes used as a mood stabilizer in the treatment of bipolar disorders. It has no known major teratogenic effects, although there is a possible risk of increases in oral clefts, such as cleft lip and cleft palate. Lithium (C) is classically associated with cardiovascular abnormalities following first-trimester exposure, such as the Ebstein anomaly (defect with the tricuspid valve). Olanzapine (D) is a second-generation antipsychotic medication used to treat a variety of psychiatric conditions. Available data have not demonstrated an increased risk of any major congenital abnormalities in the children of women taking olanzapine during pregnancy. Carbamazepine is more likely than lamotrigine, lithium, and olanzapine to cause neural tube defects, such as spina bifida. Bipolar II disorder is diagnosed in patients with at least one episode of major depression and at least one episode of hypomania. It is distinguished from bipolar I disorder by the absence of manic episodes. Hypomanic episodes present with features that are similar to manic episodes but less severe. Hypomanic episodes are characterized by an abnormal and distinct period of elevated or irritable mood with increased energy and activity. The episode must last at least 4 days, and the symptoms must be present most of the day nearly every day during this period. Patients with an elevated mood require three associated symptoms, while patients with an irritable mood require four associated symptoms. These associated symptoms include inflated self-esteem, decreased need for sleep, increased talkativeness, distractibility, racing thoughts (may manifest as flight of ideas), increased goal-directed activity, and risky behavior. The diagnosis of bipolar II disorder is made according to the criteria in the fifth edition of the Diagnostic and Statistical Manual of Mental Disorders. The symptoms must not be due to the physiologic effects of a substance. In addition, if the symptoms include psychotic features or require hospitalization, the episode is considered a manic episode. The management of bipolar II disorder includes treating hypomanic and depressive mood episodes and establishing maintenance treatment between episodes. The recommended treatment of hypomanic episodes is similar to mild or moderate manic episodes. Olanzapine and risperidone, which are atypical antipsychotics, are the first-line treatments. Second-line options include lithium, other antipsychotics, and antiseizure medications, such as valproate and carbamazepine. When possible, the treatment selection should be guided by the patient's previous response to therapies. Patients with bipolar II disorder who present with a depressive episode and who are not currently being treated with antimanic medications (lithium, valproate, carbamazepine, and lamotrigine) are treated initially with antipsychotic medications, specifically quetiapine or lurasidone. Patients who present with a depressive episode who are treated with antimanic medications should have an antipsychotic medication added to the regimen. The recommended maintenance treatment for patients with bipolar II disorder is the same regimen that led to improvement in the previous mood episode. However, if this regimen is not tolerated due to adverse effects, then lithium, valproate, quetiapine, or lamotrigine is preferred. However, valproate should ideally not be used in women of childbearing age due to its teratogenic effects. Teratogenic effects should be considered in women of childbearing age with bipolar disorder. Valproate is associated with an increased risk of neural tube defects, major malformations, and neurodevelopmental abnormalities. The major malformations can include craniofacial defects (oral clefts), cardiovascular malformations, and limb malformations (club foot and polydactyly). Children exposed to valproate in utero have lower IQ scores than the general population. Carbamazepine is associated with neural tube defects (spina bifida) and similar major malformations to those found with valproate use. However, the risk of neural tube defects is higher with exposure to valproate than carbamazepine.

A patient under your care with a history of symptoms, including hypervigilance, insomnia, nightmares, flashbacks, and an exaggerated startle response, after being involved in a car collision 3 months ago presents requesting help managing their symptoms. Which of the following is the best initial treatment approach? A. Cognitive behavioral therapy B. Cognitive behavioral therapy with fluoxetine C. Dialectical behavior therapy D. Fluoxetine

A. Cognitive behavioral therapy Cognitive behavioral therapy with fluoxetine (B) or lone treatment with fluoxetine (D) are approaches used to treat patients who are refractory to psychotherapy alone or who express a desire for pharmacotherapeutic treatment. While these approaches can be efficacious, psychotherapy alone is the recommended first-line treatment. Dialectical behavior therapy (C) is a specific type of psychotherapy most commonly used to treat patients with borderline personality disorder and can be effective in treating patients with post-traumatic stress disorder who are comorbid with this condition.

A 12-year-old patient presents to the clinic for help with aggression toward parents and family pets, frequent violations of the law that include arson and theft, and bullying others at school. Which of the following is also most likely to be true of this patient? A. Comorbid attention-deficit/hyperactivity disorder B. Comorbid obsessive-compulsive disorder C. Female sex D. Grandiosity

A. Comorbid attention-deficit/hyperactivity disorder Conduct disorder is a behavioral disorder characterized by aggression toward people or animals, destruction of property, deceitfulness, theft, and serious violation of societal norms. Diagnosis requires a persistent and repetitive pattern of such behavior over a 12-month period. Risk factors for the development of conduct disorder include poverty, turbulent home life, child abuse, male sex, parental substance use, parental criminal behavior, changes in guardian figures during childhood, peers who participate in criminal activities, harsh parental discipline, and maternal smoking during pregnancy. Conduct disorder is often comorbid with oppositional defiant disorder, attention-deficit/hyperactivity disorder, and substance use disorders. Treatment of conduct disorder begins with treatment of any comorbid disorders, specifically attention-deficit/hyperactivity disorder. Psychosocial therapy is the most effective treatment modality for conduct disorder, including counseling for both the parents and the child. School-based interventional programs with a home component are also viable therapy options. Comorbid obsessive-compulsive disorder (B) is not associated with conduct disorder, although oppositional defiant disorder is. Female sex (C) is protective, not a risk factor. Grandiosity (D) is defined as an unrealistic sense of superiority over others and is not characteristic of conduct disorder. Pediatric patients with bipolar disorder may erroneously receive a diagnosis of conduct disorder during periods of mania, at which time they may indulge in risk-taking behaviors. However, patients with bipolar disorder often manifest grandiosity as well. Other characteristics of bipolar disorder that are not often present in patients with conduct disorder and can help differentiate the two include euphoria, decreased need for sleep, hallucinations, and delusions.

A 25-year-old man presents to the clinic with irritability 5 days after discontinuing a psychoactive substance containing delta-9-tetrahydrocannabinol. Which of the following is the most common manifestation of the withdrawal syndrome associated with this substance? A. Disturbed sleep B. Lacrimation C. Piloerection D. Seizures

A. Disturbed sleep Lacrimation (B) and piloerection (C) are clinical manifestations associated with opioid withdrawal. The other manifestations of opioid withdrawal include gastrointestinal distress (abdominal cramps, diarrhea, nausea, and vomiting), flu-like symptoms (rhinorrhea, diaphoresis, and shivering), sympathetic and central nervous system arousal (mydriasis, mild hypertension and tachycardia, anxiety and irritability, insomnia, agitation, restless legs syndrome, general restlessness, and tremor), yawning, sneezing, anorexia, dizziness, myalgias, and leg cramps. Seizures (D) are a severe manifestation of withdrawal from alcohol, benzodiazepines, or barbiturates. Other symptoms can include anxiety, tremors, perceptual disturbances, dysphoria, psychosis, and autonomic instability. Which other medications have efficacy in treating patients with only sleep-related disturbances after cessation of cannabis use? -Zolpidem and nitrazepam

A 66-year-old man with a history of hyperlipidemia, type 2 diabetes, and diabetic neuropathy presents for mood disturbance for the last 3 weeks. He reports feeling his life is worthless and has lost interest in activities that used to bring him joy. He has no interest in seeing friends or family and has recurrent thoughts of death, although he has no specific suicidal plan. He reports a 5 lb unintentional weight loss but states he has lost his appetite. He mentions he has not been controlling his glucose over the last 6 months and thinks his neuropathy has worsened. Which of the following medications would be best for this patient? A. Duloxetine B. Gabapentin C. Pregabalin D. Sertraline

A. Duloxetine Gabapentin (B) is an antiepileptic medication that would not be effective in treating major depressive disorder and may only be effective at relieving the neuropathic pain. Pregabalin (C) is similar to gabapentin and may be used to treat neuropathic pain but would be less effective in treating major depressive disorder. Sertraline (D) is used as a first-line treatment for major depressive disorder but is not used in treating diabetic neuropathy. Major depressive disorder is characterized by depressed mood, loss of interest in daily activities, sleep disturbance, weight changes, psychomotor symptoms, fatigue, thoughts of worthlessness, difficulty concentrating, and recurrent thoughts of death. At least five of these symptoms must be present for at least 2 weeks with at least one symptom being either depressed mood or anhedonia to diagnose major depressive disorder. These symptoms have a significant impact on the quality of life and interpersonal relationships. Selective serotonin reuptake inhibitors (SSRIs) are typically used as first-line treatment, but the selection of a medication should consider the side effect profile and the specific patient. Serotonin and norepinephrine reuptake inhibitors (SNRIs) are effective in treating unipolar major depression. These medications work by blocking presynaptic serotonin and norepinephrine transporter proteins, thereby increasing the available serotonin and norepinephrine in the synapse. Duloxetine is a first-line treatment for neuropathic pain. Duloxetine is absorbed by the gastrointestinal tract and metabolized by the liver. This drug is a moderately potent inhibitor of the cytochrome P450 2D6 enzyme. Patients with kidney disease or hepatic impairment should generally not use duloxetine. Common side effects include nausea, dry mouth, constipation, and dizziness.

A 52-year-old man presents to the clinic for a wellness exam. Which of the following is the most appropriate frequency for screening this patient for unhealthy alcohol use? A. Every 12 months B. Every 24 months C. Every 6 months D. Every 60 months

A. Every 12 months Alcohol Use Disorder, DSM-5 Dx Problematic pattern of use leading to significant impairment or distress as manifested by at least 2 of 11 possible criteria, such as craving, unsuccessful efforts to control use, resultant physical or psychological problems, tolerance, and withdrawal Combines prior alcohol abuse and alcohol dependence Specifiers for severity, remission status, in a controlled environment The United States Preventive Services Task Force recommends all adults in primary care who have services available for follow-up should be screened annually to identify unhealthy alcohol use. Those with unhealthy use should be counseled on appropriate use. There are several methods of screening available. Some of the most commonly used methods include single-item screening, the Alcohol Use Disorder Identification Test-Consumption (AUDIT-C), and the AUDIT. Single-item screening has the benefit of being brief and easy to memorize and score. One example is asking, "How many times in the past year have you had five (for men, four for women) or more drinks in a day?" The screening test is positive when the response is one or more. The AUDIT-C is a screening test that consists of three questions aimed at assessing the frequency and quantity of alcohol use. The AUDIT is another screening test that has 10 items. Laboratory testing is generally not recommended for unhealthy alcohol use due to the additional cost and because it only detects recent alcohol use

A patient with major depressive disorder is refractory to both first- and second-line therapies. You decide to prescribe selegiline. Which of the following is appropriate patient education regarding this agent? A. Foods that contain tyramine should be strictly eliminated while taking this agent B. Leafy green vegetable intake should be regulated to maintain level serum concentration levels C. Sexual dysfunction is a common side effect D. This agent should be taken concurrently while decreasing the most recent antidepressant dosage

A. Foods that contain tyramine should be strictly eliminated while taking this agent Leafy green vegetable intake should be regulated to maintain level serum concentration levels (B) in patients who are taking warfarin. This statement does not apply to patients taking an MAOI. Sexual dysfunction is a common side effect (C) with many antidepressants but has not been noted as frequently with selegiline and other MAOIs. This agent should be taken concurrently while decreasing the most recent antidepressant dosage (D) is an inaccurate statement. Patients beginning any MAOI agent should completely discontinue their prior antidepressants for at least 2 weeks before initiating MAOI treatment.

A 23-year-old woman with no significant prior medical history presents to the clinic reporting a chronic history of sleep issues. She states that in the evenings and during attempted sleep, she often feels a persistent "crawling and tingling" feeling in the legs that is relieved by movement, making falling asleep difficult. Physical exam is benign. Which of the following is the most likely next step in diagnosis? A. Iron studies B. Magnetic resonance imaging of the brain C. Polysomnography D. Urine drug screen

A. Iron studies Restless legs syndrome is a common sleep-related movement disorder characterized by an uncomfortable or unpleasant sensation in the limbs relieved by movement. Symptoms typically occur in the evenings or at night, during periods of inactivity. Most patients with restless legs syndrome demonstrate periodic limb movements during sleep, which may or may not cause arousal from sleep. Restless legs syndrome is common, occurring in up to 15% of the population. There is a strong genetic component. Restless legs syndrome may occur as a primary, idiopathic disorder or secondary to Parkinson disease, pregnancy, iron deficiency anemia, or diabetic or uremic peripheral neuropathy. While the pathophysiology of restless legs syndrome is not completely understood, there is a strong association with reduced iron stores. Additionally, abnormalities in dopaminergic systems, circadian physiology, thalamic functions, and other neurotransmitter irregularities, including glutamate and gamma-aminobutyric acid (GABA), may be seen. The hallmark symptom of restless legs syndrome is an uncomfortable urge to move the legs, and occasionally the arms, to relieve a feeling of deep creeping, crawling, tingling, or electric sensation in the limbs. In addition to movement, patients may also report that stretching, rubbing, or hot or cold water may relieve symptoms. Restless legs syndrome may result in sleep-onset insomnia, nocturnal awakenings, depression, and anxiety. Diagnosis of restless legs syndrome is often made clinically, and the only testing that is always indicated is iron studies due to the strong association with decreased ferritin stores. The International Restless Legs Syndrome Study Group has developed diagnostic criteria for restless legs syndrome that consists of patients having all five of the following: an urge to move the legs that may or may not be accompanied by an uncomfortable sensation in the legs, the urge to move or uncomfortable sensations occur during periods of rest or inactivity, the urge to move or unpleasant sensations are partially or totally relieved by movement as long as the activity continues, the urge to move or unpleasant sensations are worse in the evening or at night than during the day, and symptoms are not solely accounted for by another medical or behavioral condition, such as leg cramps or habitual foot tapping. Polysomnography is not required for the diagnosis of restless leg syndrome, but may be helpful in cases of severe movements causing frequent sleep arousals and in ruling out other disorders, such as obstructive sleep apnea. Treatment for restless legs syndrome consists of oral iron therapy in all patients with iron deficiency or in those with low-normal ferritin levels (ferritin < 75 mcg/L). It is also important that, if possible, patients avoid aggravating drugs, such as caffeine, most antidepressants, antipsychotics, dopamine-blocking antiemetics, and centrally-acting antihistamines, and participate in daily exercise to lessen symptoms. In patients with persistent symptoms despite the aforementioned treatments, a nonergot dopamine agonist (pramipexole, ropinirole, rotigotine) or an alpha-2-delta calcium channel ligand (gabapentin, pregabalin) may be indicated.

A 32-year-old man presents to the clinic with recurrent, episodic intense fear with symptoms of choking and dyspnea. He cannot identify a trigger to these episodes but states they resolve spontaneously after 10 to 15 minutes. Which of the following comorbidities is the most likely to be found in this patient? A. Major depressive disorder B. Obsessive-compulsive disorder C. Social anxiety disorder D. Specific phobia

A. Major depressive disorder Obsessive-compulsive disorder (B) presents with obsessions that may be similar to panic attacks but are only associated with the underlying object of obsession and compulsion. Social anxiety disorder (C) manifests with fear of specific social situations, which can trigger a panic attack. Similarly, panic attacks associated with specific phobia (D) are caused by circumscribed phobic objects or situations. These conditions are important to rule out prior to diagnosing panic disorder, in which panic attacks are unexpected and have no known associated trigger. While these conditions may be comorbid, they are less likely to be present than major depressive disorder. Panic disorder is an anxiety disorder characterized by unexpected, sudden onset, overwhelming fear, which is termed panic attacks. These panic attacks are accompanied by somatic and cognitive symptoms, including racing thoughts, a choking sensation, racing heartbeat, sweating, chest pain, and numbness, among others. The median age of onset for panic disorder is 24 years old, with a prevalence twice as high in women versus men. Approximately 37% of patients with panic disorder also have a lifetime prevalence of major depressive disorder. The Diagnostic and Statistical Manual of Mental Disorders, fifth edition diagnostic criteria for panic disorder include recurrent, unexpected panic attacks followed by at least 1 month of anticipatory concern or worry about additional attacks, maladaptive behavioral changes related to the attacks, or both. Other causes of the panic attacks must be ruled out prior to the diagnosis of panic disorder. Common conditions in the differential diagnosis include social anxiety disorder, specific phobia, obsessive-compulsive disorder, and post-traumatic stress disorder. Initial treatment of panic disorder includes either cognitive behavioral therapy or antidepressants. Pharmacotherapy approaches initially involve selective serotonin reuptake inhibitors (e.g., fluoxetine, sertraline) for at least 4 weeks. If the patient is refractory to this treatment strategy, an alternative selective serotonin reuptake inhibitor or a serotonin and norepinephrine reuptake inhibitor should be implemented. Patients who do not respond to pharmacotherapy approaches should be started on psychotherapy, specifically cognitive behavioral therapy. Benzodiazepines (e.g., lorazepam, diazepam) can be used to augment antidepressant treatment in those with a partial response after 10 to 12 weeks of treatment. Treatment-resistant panic disorder in patients without substance or alcohol use disorders can be treated with long-acting benzodiazepines (e.g., clonazepam) either adjunctively or as a monotherapy. Alternatively, gabapentin, pregabalin, or mirtazapine is the preferred agent in patients with concomitant substance or alcohol use disorders Which serotonin and norepinephrine reuptake inhibitor is approved for the treatment of panic disorder? Answer: Venlafaxine.

A 22-year-old man presents with "hearing voices." He has had progressively worsening symptoms over the last 7 months. He has heard voices saying negative statements that seem to come from inside of his head. He has also had a belief that his neighbors are trying to poison him and have planted cameras and listening devices inside his house. When speaking about these symptoms, he becomes distracted easily and gets off topic quickly. He is also speaking quickly and is very talkative. He notes he has not slept in 2 days but does not feel tired. He reports no substance use or use of other medications. What is the most likely diagnosis? A. Schizoaffective disorder B. Schizoid personality disorder C. Schizophrenia D. Schizophreniform disorder

A. Schizoaffective disorder Schizoaffective disorder is a psychotic disorder that is associated with significant impairment in functional capabilities. Characteristics include positive and negative symptoms, cognitive impairment, and mood disturbances. Positive symptoms include hallucinations and delusions. Disorganized thought content and processes are also a manifestation of a positive symptom and can include tangential or circumstantial speech and derailment. Negative symptoms are defined as the absence of normal behavior and include decreased expression, apathy, and a flat affect. Cognitive impairment usually affects memory, learning abilities, attention, and processing speed. Diagnosis is often one of exclusion of other differential diagnoses. The delusions and hallucinations specific to schizoaffective disorder are called first-rank symptoms. These symptoms include audible hallucinations, thought insertion or withdrawal, and audible self-thoughts. Diagnosis is made when two or more of the characteristic symptoms are present for at least 1 month and significant impairment is present for at least 6 months. Schizoaffective disorder is distinguished from schizophrenia by the presence of manic episodes or a significant depressive episode in addition to the symptoms listed above. Schizoid personality disorder (B) is characterized by the absence of normal behavior and social relationships in a long-standing and pervasive pattern. There is no psychosis with this disorder. Schizophrenia (C) is similar to schizoaffective disorder, but there are no manic or depressive episodes associated with schizophrenia. Schizophreniform disorder (D) is similar to schizoaffective disorder, but the total duration of the disorder is < 6 months and there is no mania associated with schizophreniform disorder.

Which antihypertensive medications are sometimes used in the treatment of post-traumatic stress disorder?

Alpha-blockers and clonidine Post-traumatic stress disorder is characterized by a prolonged psychologic and physiologic response to a traumatic experience. The response is somatic, behavioral, cognitive, and affective, involving reliving the inciting incident, flashbacks, nightmares, impaired sleep, hypervigilance, and avoiding triggers. Triggers may include people, places, or situations that bring the traumatic event to mind. The inciting traumatic experience is most often sexual relationship violence but may also include the death or life-threatening illness of a loved one, interpersonal violence, participation in or exposure to group violence, or other life-threatening events, such as a natural disaster, motor vehicle collision, or prolonged hospital stay. Predisposing factors in the development of post-traumatic stress disorder include young age at time of trauma; adverse childhood events; female sex; being widowed, separated, or divorced; lower socioeconomic status; lower education level; personal or family history of psychiatric disease; poor social support; and a severe stress reaction at the time of the inciting event. Treatment of post-traumatic stress disorder should start with trauma-focused psychotherapy with exposure and may also include selective serotonin reuptake inhibitors. Post-Traumatic Stress Disorder (PTSD) Sx duration > 1 month Persistently reexperiencing of event Persistently ↑ arousal Avoidance of stimuli ↑ risk for suicide, substance use

A 20-month-old boy presents to the pediatrician with his mother, who is concerned that he does not seem interested in interacting with her. She has also noticed that he does not like to make eye contact and has intense interests in visual stimuli, such as flashing lights. He seems easily irritated by small changes in the daily routine. At which of the following months does the American Academy of Pediatricians recommend screening for the suspected diagnosis? A. 12 and 18 months B. 18 and 24 months C. 24 and 30 months D. 30 and 36 months

B. 18 and 24 months 12 and 18 months (A), 24 and 30 months (C), and 30 and 36 months (D) are incorrect. The American Academy of Pediatrics recommends screening for autism spectrum disorder at 18 and 24 months.

A 38-year-old man presents to the emergency department with his wife. The patient is in an altered mental state and continues to report there are spiders crawling on him. His vitals are heart rate of 138 bpm, blood pressure of 158/96 mm Hg, and respiratory rate of 20 breaths per minute. He appears angry and diaphoretic on exam. The patient's wife reports heavy alcohol use at home. Which of the following timeframes is consistent with this patient's presenting symptoms? A. 12-48 hours after the last drink B. 48-96 hours after the last drink C. 6-24 hours after the last drink D. 6-48 hours after the last drink

B. 48-96 hours after the last drink The patient in the above vignette is going through alcohol withdrawal, specifically delirium tremens, which is an acute onset of confusion that occurs between 48-96 hours after the patient's last alcoholic drink. Risk factors include a history of drinking, a history of previous alcohol withdrawal seizures, a history of delirium tremens, and age > 30 years old. Delirium tremens presents as fluctuating disturbance of attention and cognition, sometimes with f hallucinations, disorientation, tachycardia, hypertension, agitation, fever, and diaphoresis after discontinuing alcohol. Hypomagnesemia is also common in patients with delirium tremens, which increases their risks for seizures and dysrhythmias. Delirium tremens is considered a medical emergency. Treatment is with immediate administration of IV benzodiazepines, such as diazepam or lorazepam, along with IV fluids and supplements, such as thiamine, folic acid, and multivitamins. 12-48 hours after the last drink (A) a patient may develop alcoholic hallucinations with normal and stable vitals. Although the patient in the above vignette is having hallucinations, this time period is not appropriate because the patient's vitals are unstable and he is experiencing delirium tremens. 6-24 hours after the last drink (C) symptoms of alcohol withdrawal begin. Patients may present with symptoms of increased CNS activity, such as tremors, anxiety, sweating, palpitations, and GI upset. These are all appropriate symptoms for an uncomplicated alcohol withdrawal presentation. Withdrawal seizures usually occur 6-48 hours after the last drink (D). The patient in the above vignette is not experiencing a seizure and thus this timeframe is not appropriate.

A 24-year-old woman presents to her therapist to discuss her dietary habits. She reports that she intermittently consumes large amounts of food, such as two large pizzas or two dozen cookies, and subsequently feels guilty. Which of the following additional features is most suggestive of binge eating disorder? A. Abnormally low body weight B. Absence of compensatory behaviors C. Distortion of body weight D. Excessive exercise

B. Absence of compensatory behaviors Abnormally low body weight (A) is incorrect. Patients with binge eating disorder are typically overweight. Distortion of body weight (C) is an important feature of anorexia nervosa. It occurs when individuals have a view of their body image that is significantly different from reality. Excessive exercise (D) is an example of a compensatory behavior. The presence of compensatory behaviors is one of the key factors that distinguishes bulimia nervosa from binge eating disorder. Binge Eating Disorder Eating distinctly more than most people within a certain time period Lack of control, overeating, marked distress Tx: CBT is first line, weight management if also needed, medication options include lisdexamfetamine, topiramate, SSRIs Binge eating disorder is an eating disorder in which individuals have episodes of binge eating without compensatory behaviors. Binge eating is defined as consuming an amount of food in a discrete period of time that is larger than what most people would eat in a similar amount of time under similar circumstances. Binge eating episodes are also characterized by the following features: having a sense of loss of control, eating rapidly, eating until feeling uncomfortably full, eating when not physically hungry, eating alone due to embarrassment regarding the amount of food consumed, and feeling distress about overeating. The episodes must occur at least once per week for at least 3 months. Binge eating disorder is more common in women than men and typically begins during adolescence or young adulthood. Binge eating disorder is diagnosed according to the criteria in the fifth edition of the Diagnostic and Statistical Manual of Mental Disorders. Psychotherapy is the recommended first-line treatment for the disorder, and cognitive behavioral therapy is the recommended psychotherapy. Behavioral weight loss therapy is recommended in patients who either do not improve with psychotherapy or cannot try psychotherapy. The purpose of behavioral weight loss therapy is to improve behaviors by reducing caloric restriction, increasing physical activity, and improving nutrition. Pharmacologic treatment is another second-line treatment modality that can be used as monotherapy or to augment cognitive behavioral therapy. Selective serotonin reuptake inhibitors, such as sertraline, are the preferred pharmacologic class of medications.

A 24-year-old man presents to a therapist because he lacks confidence in social settings. Despite having a desire for close relationships, he states that, for years, he has avoided most social situations due to a fear of embarrassment. Which of the following personality disorders is the most likely diagnosis? A. Antisocial B. Avoidant C. Dependent D. Schizoid

B. Avoidant Antisocial (A) personality disorder is marked by a persistent pattern of exploitation of others and breaking rules. Individuals with antisocial personality disorder lack empathy for others and remorse for their actions. Dependent (C) personality disorder is characterized by an excessive need to be taken care of by others and difficulty with decision-making. It is also a cluster C personality disorder. Schizoid (D) personality disorder is marked by a persistent pattern of social withdrawal due to a lack of interest in close relationships. Individuals with schizoid personality disorder are often considered to be eccentric and reclusive. Avoidant personality disorder is marked by feelings of inadequacy, hypersensitivity to criticism or embarrassment, and avoidance of social situations. It is one of the cluster C personality disorders, which also includes dependent personality disorder and obsessive-compulsive personality disorder. Individuals with avoidant personality disorder avoid social interactions despite having a desire for companionship. This contrasts with schizoid personality disorder, in which individuals avoid social interactions because they do not desire close relationships. Individuals with avoidant personality disorder typically choose occupations with limited social interactions. Social anxiety disorder is a common comorbidity of individuals with avoidant personality disorder. It occurs with equal frequency in men and women. The fifth edition of the Diagnostic and Statistical Manual of Mental Disorders (DSM-5) diagnostic criteria for avoidant personality disorder include a pervasive pattern of social inhibition, feelings of inadequacy, and hypersensitivity to negative evaluation, which begin by early adulthood and present in a variety of contexts. Psychotherapy with social skills training is the most effective treatment of avoidant personality disorder.

A 22-year-old woman with obesity presents to the clinic for episodes in which she eats abnormally large portions of food. She reports feeling a lack of control during these episodes and feels guilty afterward about her eating. The patient reports no self-induced vomiting and is not fasting, taking medications to lose weight, or participating in extreme exercise. These episodes have been occurring about five times per week for the past year. Which of the following is the most likely diagnosis? A. Anorexia nervosa B. Binge eating disorder C. Bulimia nervosa D. Prader-Willi syndrome

B. Binge eating disorder Anorexia nervosa (A) is incorrect. One of its key features is restricted caloric intake that leads to an abnormally low body mass index. The patient in this vignette has obesity with episodes of binge eating. Furthermore, patients with anorexia nervosa often have compensatory behaviors to prevent weight gain. Patients with either disorder can have an intense fear of gaining weight. Bulimia nervosa (C) is distinguished from binge eating disorder by the presence of compensatory mechanisms, such as purging, laxative use, diuretic use, starvation, and excessive exercise, to prevent weight gain following binge eating episodes. Prader-Willi syndrome (D) is a genetic disorder that results in obesity. However, it often presents during infancy with neonatal hypotonia. Infants often have difficulty feeding, which may cause failure to thrive. Toddlers typically have delays in major developmental milestones. Children with Prader-Willi syndrome typically have short stature and develop obesity if access to food is not restricted. Prader-Willi syndrome is unlikely to be the cause of this patient's presentation because the onset of the binge eating episodes began during adulthood. Treatment options include psychotherapy (preferably cognitive behavioral therapy), behavioral weight loss therapy, and pharmacotherapy. Psychotherapy is the preferred first-line treatment. Behavioral weight loss therapy is recommended for patients who do not have access to psychotherapy, decline psychotherapy, or do not improve with psychotherapy. It consists of improving lifestyle measures to promote weight loss, such as implementing moderate caloric restriction, increasing aerobic exercise, and improving nutrition. There are several pharmacologic agents that may be used for the treatment of binge eating disorder. Selective serotonin reuptake inhibitors, such as fluoxetine, are often first line. However, topiramate, zonisamide, and lisdexamfetamine may also be used.

A 27-year-old man presents to the emergency department with acute agitated delirium. His girlfriend states he began an increased dose of duloxetine earlier today. Vital signs are notable for a heart rate of 135 bpm and blood pressure of 220/115 mm Hg. Physical examination reveals diaphoresis of the skin and mydriasis. Which of the following additional findings supports the suspected diagnosis? A. Bradyreflexia B. Inducible ankle clonus C. Moist mucous membranes D. Recent initiation of atropine

B. Inducible ankle clonus Serotonin syndrome is a life-threatening condition due to increased serotonergic activity in the central nervous system. It typically occurs within 24 hours of initiating or increasing the dose of a serotonergic agent. It most commonly occurs when two or more serotonergic drugs are combined. Selective serotonin reuptake inhibitors (e.g., sertraline and fluoxetine) or serotonin and norepinephrine inhibitors, such as duloxetine, are common pharmacologic causes. Illicit drugs, such as cocaine or amphetamines, are also common causes. The classic triad associated with serotonin syndrome includes mental status changes, neuromuscular excitation, and sympathetic hyperactivity. Patients are often agitated and anxious. Neuromuscular excitation may manifest as clonus (most often inducible ankle clonus), deep tendon hyperreflexia, tremors, akathisia, and hypertonia. Clonus is a classic and sensitive finding for serotonin syndrome. Sympathetic hyperactivity may manifest as tachycardia, hypertension, skin flushing, diaphoresis, dilated pupils, and dry mucous membranes. Hyperthermia is seen in severe cases. Complications may include cardiac dysrhythmias, seizures, rhabdomyolysis, and metabolic acidosis. Serotonin syndrome is diagnosed clinically. The differential diagnosis includes alcohol withdrawal, anticholinergic toxicity, sepsis, thyrotoxicosis, and neuroleptic malignant syndrome. Discontinuing all serotonergic agents and providing supportive care are the main principles of the management of serotonin syndrome. Benzodiazepines are often used to sedate patients. Serotonin antagonists, such as cyproheptadine, are recommended for patients whose vital signs and agitation are not improving with supportive care and benzodiazepines. Patients with severe hyperthermia require sedation, paralysis, and intubation to reduce muscle activity. Antipyretic agents, such as acetaminophen, are not effective in treating patients with hyperthermia due to serotonin syndrome Serotonin syndrome causes hyperreflexia. Neuroleptic malignant syndrome causes bradyreflexia (A). Serotonin syndrome causes dry mucous membranes (C). Atropine (D) is an anticholinergic medication, which could support a diagnosis of anticholinergic toxicity.

A 25-year-old man with a history of recurrent major depressive episodes presents to his psychiatrist with racing thoughts for the past 5 days. He had a similar episode a month ago that lasted 6 days. He describes his mood as euphoric and states he has increased confidence in himself. He reports he has only needed to sleep around 3 hours and has spent hours on the phone catching up with old friends. He also reports plans to start an automotive repair business out of his garage and has created social media advertisements. He reports no drug use. Which of the following is the most likely diagnosis? A. Bipolar I disorder B. Bipolar II disorder C. Borderline personality disorder D. Narcissistic personality disorder

B. Bipolar II disorder Bipolar I disorder (A) is diagnosed in patients who have had a manic episode. Manic episodes and hypomanic episodes have similar clinical features but are distinguished by the severity of the episode. Any episode that has psychotic features (delusions or hallucinations) or is severe enough to require hospitalization is a manic episode. Manic episodes markedly impair an individual's ability to function, whereas hypomanic episodes either improve function or mildly impair function. The diagnostic criteria for a manic episode require a symptom duration of 7 days, whereas the criteria for a hypomanic episode only require 4 days of symptoms. Borderline personality disorder (C) and narcissistic personality disorder (D) are each marked by pervasive personality patterns rather than an acute episode. They are cluster B personality disorders. The core features of narcissistic personality disorder are grandiosity, hypersensitivity to negative feedback, exaggeration of accomplishments, lack of empathy, and lack of consideration of others. The core features of borderline personality disorder are unstable mood, behavior, and interpersonal relationships; fear of abandonment; intense attachment; suicidal behavior; and poorly formed identity.

A 52-year-old woman presents with concerns for mood changes. She states she has had feelings of hopelessness and has been having difficulty sleeping for the last month. She has lost weight due to a poor appetite and has lost interest in daily activities. She has a medical history significant for a seizure disorder and controlled hypertension. Which of the following medications would be best for this patient in addition to psychotherapy? A. Amitriptyline B. Citalopram C. Phenelzine D. Venlafaxine

B. Citalopram Unipolar major depression is characterized by depressed mood, loss of interest in daily activities, sleep disturbance, weight changes, psychomotor symptoms, fatigue, thoughts of worthlessness, difficulty concentrating, and recurrent thoughts of death. There must be at least five of these symptoms present for at least 2 weeks with at least one symptom being either depressed mood or anhedonia to diagnose unipolar major depression. These symptoms cause a significant impact on the quality of life and interpersonal relationships. Assessment should include laboratory testing to rule out physiologic and pathologic causes of depression. Treatment goals should focus on restoring baseline function and the ability to participate in activities of daily living without impairment. Initial therapy should include psychotherapy and pharmacological therapy. Many antidepressants have demonstrated efficacy in treating unipolar major depression. Selective serotonin reuptake inhibitors (SSRIs) are usually used as first-line treatment, but the selection of a medication should consider the side effect profile and the specific patient. Citalopram is an SSRI that has a side effect profile that includes headache, nausea, nervousness, sexual dysfunction, and weight gain. Despite these side effects, SSRIs are generally well tolerated, although at high doses, they may cause QT prolongation. Amitriptyline (A) is a tricyclic antidepressant that may lower the seizure threshold, making this an unsuitable choice for an individual with a history of seizure disorder. Phenelzine (C) is a monoamine oxidase inhibitor and is not used first line due to the extensive side effect profile and dietary restrictions associated with this drug. Venlafaxine (D) is a serotonin and norepinephrine reuptake inhibitor that can cause dose-related hypertension and requires monitoring of blood pressure while taking this medication.

A 54-year-old man presents to the emergency department after being found unconscious in the street. He has a history of intravenous heroin use and was given naloxone en route to the emergency department. The patient is alert, irritable, and restless on presentation. Vital signs include a temperature of 98.6°F, heart rate of 115 bpm, and blood pressure of 165/94 mm Hg. You notice pupil dilation, yawning, and piloerection during physical examination. Which of the following is the recommended treatment? A. Buprenorphine B. Clonidine C. Methadone D. Morphine

B. Clonidine Buprenorphine (A) is a partial agonist of opioid receptors. Partial agonists bind to the receptors but only have a partial effect on the receptor compared to a full agonist. Buprenorphine is one of the opioid replacements that can be administered to patients who have an unwanted interruption in opioid use. Methadone (C) is a long-acting opioid receptor agonist used for opioid replacement therapy. It has a long duration of action, which helps it to prevent opioid withdrawal symptoms. Morphine (D) is a naturally occurring opioid used to provide analgesia. It is not used to treat opioid withdrawal due to its short duration of action.

A 34-year-old man who works as a neurosurgeon presents to a therapist with his wife for marital counseling. His wife discusses that he has a high sense of self-importance and is hypersensitive to negative evaluation by others. She also reports that he continuously brags about his professional accomplishments and lacks empathy for others who are not as successful. Which cluster of personal disorders does this patient's diagnosis most likely belong to? A. Cluster A B. Cluster B C. Cluster C D. Cluster D

B. Cluster B Narcissistic personality disorder is one of the cluster B personality disorders, along with borderline, antisocial, and histrionic personality disorders. It is more common in men, and it typically begins in early adulthood. The core features include grandiosity (sense of high self-importance), hypersensitivity to evaluation, exaggeration of accomplishments, exploitative relationships, a lack of empathy, and a lack of consideration for others. Most individuals with narcissistic personality disorder do not have insight into their narcissism. The diagnosis of narcissistic personality disorder is made based on clinical criteria found in the fifth edition of the Diagnostic and Statistical Manual of Mental Disorders. The core clinical features must be present by early adulthood and in a variety of contexts. Psychotherapy is the recommended treatment for narcissistic personality disorder. Cluster A (A) includes schizoid, paranoid, and schizotypal personality disorders. Cluster C (C) includes dependent, avoidance, and obsessive-compulsive personality disorders. Cluster D (D) is not one of the clusters of personality disorders.

A 23-year-old man presents to the clinic with concerns that he is unable to form relationships with others. He notes that he wants to make friends with his co-workers, but he is always scared to attend social gatherings and happy hours after work. When you ask why the patient is hesitant to attend these events, he reports feeling like he may not be liked by others. Which of the following is the most appropriate intervention for this patient? A. Alprazolam as needed B. Cognitive behavior therapy C. Daily escitalopram D. Propranolol as needed

B. Cognitive behavior therapy The patient in the above vignette has avoidant personality disorder. These individuals want to develop a relationship with others but avoid them due to feeling inadequate, inferior, or having fear of rejection or humiliation. These individuals are usually awkward in public situations. According to the fifth edition of the Diagnostic and Statistical Manual of Mental Disorders (DSM-5), an individual must meet four (or more) of the following criteria to be diagnosed with avoidant personality disorder: 1) avoids activities that involve direct person-to-person contact due to fear of being humiliated, rejected, or disapproved of; 2) unwilling to interact with others unless they know for sure they will be liked; 3) shows restraint within intimate relations due to the same fear; 4) is noted to be lost in thought with being rejected during any social interaction; 5) is shy during new social interactions due to being scared of not being liked; 6) views themselves as inadequate or inferior to others; and 7) is usually hesitant to take risks or engage in a new activity due to fear of being humiliated. The best form of management for patients with avoidant personality disorder is psychotherapy, which includes both cognitive behavior therapy and social training. Pharmacological therapy can be used in adjunct to therapy. This therapy can include a beta-blocker as needed for anxiety or a selective serotonin reuptake inhibitor for associated depression. Alprazolam as needed (A) is the correct treatment for acute panic attacks. These individuals do not avoid social situations, rather their panic attacks occur at random and occur abruptly. Daily escitalopram (C) and propranolol as needed (D) are both appropriate treatments only as adjunctive therapy for those with avoidant personality disorder if they have underlying depression or anxiety. Overall, medication therapy is not the first-line treatment for any personality disorder.

A 23-year-old woman presents to the clinic due to anxiety attacks. During these attacks, her heart races, she sweats and feels like she is going to die, and she gets tightness in her chest, nausea, and dizziness. Several visits to the emergency department in the past have determined no physical cause for these symptoms. The attacks started 1 year ago and have occurred only three or four times since then, but to avoid another attack, the patient now stays at home nearly all day, every day and is unable to work. Which of the following is the most effective nonpharmacologic clinical intervention for this patient? A. Acceptance-based exposure therapy B. Cognitive behavioral therapy C. Interpersonal psychotherapy D. Relaxation therapy

B. Cognitive behavioral therapy Acceptance-based exposure therapy (A) is a method of psychotherapy whereby a patient is subjected to triggers of anxiety or panic in an attempt to lessen fear of these situations. However, in this method, breathing techniques, muscle relaxation, and other tools for anxiety control are not introduced first. Interpersonal psychotherapy (C) involves addressing interpersonal conflicts that result in anxiety. Interpersonal psychotherapy is inferior to cognitive behavioral therapy in the treatment of panic disorder. Relaxation therapy (D) is a component of the cognitive behavioral approach, but when it is used alone, it is inferior to cognitive behavioral therapy in the treatment of panic disorder. Panic disorder is characterized by recurrent unexpected panic attacks with at least one of the attacks being followed by at least 1 month of either persistent worry about additional attacks or their consequences or a significant maladaptive change in behavior related to the attacks. A panic attack is a discrete episode of intense fear accompanied by four or more of the following symptoms: palpitations, diaphoresis, shaking, shortness of breath, chest pain, nausea, feelings of choking, dizziness, chills, paresthesias, derealization, fear of losing control, and fear of dying. These symptoms tend to peak within 10 minutes and then gradually subside. Panic disorder is more common in women, has a familial component, and generally begins before 25 years of age. Other psychiatric or physiologic causes of symptoms of panic attack should be ruled out before a diagnosis of panic disorder is made. Treatment for panic disorder can involve nonpharmacologic and pharmacologic therapy. The nonpharmacologic intervention with the most promising clinical data is cognitive behavioral therapy. In cognitive behavioral therapy for panic disorder, the patient is taught about the nature of the disorder and is encouraged to keep a mood diary as well as a record of any panic attacks. Next, the patient is taught breathing techniques and muscle relaxation techniques. Incorrect or maladaptive thoughts are addressed. Finally, patients are exposed repeatedly and systematically in a graduated manner to the situations that trigger panic attacks. Pharmacologic therapy for panic disorder may include selective serotonin reuptake inhibitors, serotonin and norepinephrine reuptake inhibitors, tricyclic antidepressants, or mirtazapine.

A 62-year-old man is admitted in stable condition following an uncomplicated total hip arthroplasty. His known medical history includes hypertension, peripheral vascular disease, and chronic tobacco use. After 10 hours, he reports nausea and appears anxious. The patient is diaphoretic, and a tremor is noted during intentional movement. Which of the following medications is best to initiate at this time? A. Carbamazepine B. Diazepam C. Naltrexone D. Phenobarbital

B. Diazepam Alcohol withdrawal syndrome is characterized by autonomic hyperactivity, tremors, insomnia, nausea and vomiting, transient hallucinations, psychomotor agitation, anxiety, and tonic-clonic seizures. These symptoms typically appear within 1 to 2 days of alcohol abstinence but may begin as early as 2 to 6 hours after reducing alcohol consumption. Symptom severity can range from mild to a severe and life-threatening presentation known as delirium tremens. Tremulousness is the earliest symptom of alcohol withdrawal, beginning within 6 hours of abstinence. Associated sympathetic hyperactivity symptoms include insomnia, anxiety, gastrointestinal upset, diaphoresis, and palpitations. Withdrawal seizures occur in clusters of two to six episodes within 6 to 48 hours of abstinence and are most likely to occur in those with a history of chronic alcohol use disorder. Hallucinations can present as visual, tactile, or auditory stimuli, develop within 12 hours of abstinence, and typically resolve by 48 hours. Delirium tremens, which is characterized by fluctuating levels of consciousness, cognitive disturbances, profound confusion, and severe autonomic hyperactivity, begins within 48 to 72 hours of alcohol consumption reduction and can last several days to a week. Treatment for patients with signs and symptoms of alcohol withdrawal syndrome is initiated with benzodiazepines. Long-acting agents (e.g., diazepam, chlordiazepoxide) can be given in high doses to treat symptoms with slow clearance of the drug to prevent symptom recurrence. Short-acting benzodiazepines (e.g., lorazepam) can alternatively be used as needed for symptoms. Supportive measures include adequate hydration and replacement of deficient electrolytes. The Clinical Institute Withdrawal Assessment for Alcohol—Revised (CIWA-Ar) is a clinical scale used to guide treatment once the diagnosis of alcohol withdrawal syndrome has been made. This scale requires patients to rank 10 of the most common signs and symptoms of alcohol withdrawal from a baseline score of 0 to the most severe presentation score of 7 in all categories except orientation, which is rated from 0 to 4. Values from each category are then added for a cumulative score with a score < 8 representing mild withdrawal, a score of 9 to 15 indicating moderate withdrawal, and a score > 15 indicating severe withdrawal. If administering short-acting benzodiazepines, this scoring system provides timing recommendations for the next dose of medication with no medication necessary for mild withdrawal, doses every 2 hours for moderate withdrawal, and hourly medication administration for patients with severe withdrawal scores.

A 60-year-old man presents to the clinic due to panic attacks. He previously canceled three of his office visits and is very nervous about this encounter. He states that, for the past 2 months, he mostly stays home because he is afraid he will have a panic attack in public and be embarrassed. Which of the following situations is this patient also likely to avoid? A. Being at home with a spouse B. Driving a car C. Sharing a meal with a close friend D. Speaking to a group via a video conference

B. Driving a car Agoraphobia is fear of being in a situation where help may not be readily available or escape may not be easily attained in the event of developing panic-like, embarrassing, or incapacitating symptoms. Agoraphobia is often accompanied by panic disorder, and the patient is often fearful of having a panic attack in certain situations. This fear leads to avoidant behavior and can severely limit a patient's daily activities. Patients with agoraphobia commonly avoid driving a car, flying in a plane, waiting in line, being at home alone, being away from home, going to a doctor's or dentist's office, or going to a salon or barber. Patients also often avoid shopping malls, public transportation, restaurants, movie theaters, wide open spaces, grocery stores, and crowded spaces. The mobility inventory is a useful tool that can assess the severity of a patient's agoraphobia. A score of 1.61 or greater on the avoidance alone scale indicates the presence of agoraphobia. Treatment of agoraphobia includes cognitive behavioral therapy and pharmacotherapy for panic disorder if present. Selective serotonin reuptake inhibitors are first-line pharmacotherapy for panic disorder. Being at home with a spouse (A) is often where a patient with agoraphobia feels most comfortable. In this situation, they feel assured of help and lack of embarrassment should they have a panic attack. Sharing a meal with a close friend (C) is less likely to be a situation avoided by a patient with agoraphobia than driving, especially if the meal is at home. Speaking to a group via a video conference (D) may not bother a patient with agoraphobia because they can easily escape the situation by turning off the video feed. Public speaking may not necessarily produce fear in a patient with agoraphobia, but fear of having a panic attack during the event may be present.

A 43-year-old man presents with anhedonia, insomnia, weight loss, intense fatigue, and feelings of worthlessness for the past 3 weeks. In addition to psychotherapy, you decide to prescribe the most suitable atypical antidepressant based on his symptoms. Which of the following side effects for this medication should you educate the patient about? A. Diarrhea B. Drowsiness C. Orthostatic hypotension D. Psychomotor agitation

B. Drowsiness Major depressive disorder is a widely prevalent psychiatric condition that is diagnosed based on criteria established in the fifth edition of the Diagnostic and Statistical Manual of Mental Disorders. Patients must exhibit depressed mood or anhedonia (i.e., lack of interest) with four or more additional symptoms, including insomnia or hypersomnia; unintentional weight loss or gain; psychomotor retardation or agitation; fatigue or low energy; decreased ability to concentrate, think, or make decisions; thoughts of worthlessness or inappropriate guilt; or recurrent thoughts of death or suicidal ideation with or without suicidal intent. Symptoms must occur nearly every day for at least 2 consecutive weeks. The most effective treatment strategy for major depressive disorder involves psychotherapy in conjunction with pharmacotherapy. Second-generation antidepressants used in the treatment of major depressive disorder include selective serotonin reuptake inhibitors (sertraline, citalopram), serotonin and norepinephrine reuptake inhibitors (duloxetine, venlafaxine), atypical antidepressants (mirtazapine, bupropion, agomelatine), and serotonin modulators (nefazodone, trazodone). Tricyclic antidepressants (amitriptyline, nortriptyline) and monoamine oxidase inhibitors (phenelzine, isocarboxazid) are older antidepressants that are more commonly used as second-line medications. Selective serotonin reuptake inhibitors are a common initial treatment but may be limited by side effects, such as sexual dysfunction and weight gain. Atypical antidepressants may be used as an alternative to selective serotonin reuptake inhibitors or serotonin and norepinephrine reuptake inhibitors. These include mirtazapine and bupropion, with agomelatine available outside the United States. Bupropion may cause insomnia or psychomotor agitation, while mirtazapine is most likely to result in drowsiness and weight gain. Another class of antidepressant medication, serotonin modulators, includes nefazodone, trazodone, vilazodone, and vortioxetine. Side effects for nefazodone most commonly include drowsiness and gastrointestinal toxicity, while trazodone may result in orthostatic hypotension, QTc prolongation, gastrointestinal toxicity, and severe drowsiness or somnolence. Diarrhea (A) and other gastrointestinal toxicity symptoms (e.g., nausea, vomiting) are most commonly caused by selective serotonin reuptake inhibitors. Orthostatic hypotension (C) is a known side effect of many tricyclic antidepressant medications (imipramine, amitriptyline) as well as trazodone. Psychomotor agitation (D) and insomnia are not associated with mirtazapine but can be caused by monoamine oxidase inhibitors (isocarboxazid, phenelzine) and certain selective serotonin reuptake inhibitors (sertraline, fluoxetine).

A 32-year-old woman presents to her obstetrician at her 6-week postpartum visit. She reports having a depressed mood for the past 2 weeks with anhedonia and increased guilt. Which of the following is the most widely used screening test for the suspected diagnosis? A. Beck Depression Inventory for Primary Care B. Edinburgh Postnatal Depression Scale C. Patient Health Questionnaire-9 D. World Health Organization Well-Being Index

B. Edinburgh Postnatal Depression Scale Postpartum depression is defined as major depressive disorder occurring in the first 12 months after delivery. Individuals are at increased risk of depression during this time period. The risk factors for postpartum depression include prior depression, prior postpartum depression, stressful life events, and poor social or financial support. The clinical manifestations of postpartum depression are the same as depression outside of the postpartum period: depressed mood, anhedonia (loss of pleasure), increased or decreased appetite, weight gain or weight loss, insomnia or hypersomnia, fatigue, poor concentration, psychomotor agitation, excessive guilt, and suicidal ideation. However, the mood symptoms, which include depressed mood, anhedonia, excessive guilt, and poor concentration, tend to be more specific for postpartum depression than somatic symptoms, such as changes in weight or appetite, because somatic symptoms occur in most postpartum individuals. Postpartum depression is considered severe in patients who have one or more of the following features: at least seven symptoms, suicidal ideation, or psychotic features. The United States Preventive Services Task Force recommends that all postpartum individuals be screened for postpartum depression, and screening should typically occur 4-8 weeks after delivery. The Edinburgh Postnatal Depression Scale is the recommended screening tool. It is a 10-item self-report assessment that can be completed in < 5 minutes and can be repeated later to evaluate for improvement or worsening of symptoms. Individuals who score at least 11 on the Edinburgh Postnatal Depression Scale are considered to be positive for postpartum depression. Individuals with postpartum depression are often treated with psychotherapy, antidepressants, or both. Selective serotonin reuptake inhibitors are often the first-line antidepressant. The Beck Depression Inventory for Primary Care (A) is a highly sensitive and specific test for identifying major depressive episodes in primary care patients. It is not used to screen for postpartum depression. The Patient Health Questionnaire-9 (PHQ-9) (C) is a test used to screen for depression. It has a sensitivity of 88% and a specificity of 85%, which makes it more accurate than most screening tests for depression. It can also be used to monitor a patient's response to treatment. Patients who screen positive on the PHQ-9 should be carefully assessed for depression. However, it cannot be used alone to diagnose depression. The World Health Organization Well-Being Index (D) is a highly sensitive but less specific screening tool for depression. It is not used to screen for postpartum depression.

A 25-year-old woman presents to the clinic with neck and shoulder pain. She also complains of excessive worry about her children's well-being and her husband's fidelity. She states she has frequent headaches, trouble sleeping, feelings of being overwhelmed, difficulty concentrating, and increased irritability. Vital signs are within normal limits, and complete blood count, comprehensive metabolic panel, and thyroid panel are within normal limits. Which of the following would help confirm the most likely diagnosis? A. Abnormal cervical spine X-ray B. Excessive worry has caused the patient to quit her job C. Patient Health Questionnaire-9 score of 21 D. Symptoms more days than not, lasting 2 to 3 weeks

B. Excessive worry has caused the patient to quit her job Generalized anxiety disorder is the most common psychiatric disorder encountered in primary care and describes a state of excessive worry that is difficult to control and causes significant impairment in daily functioning. Predisposing factors to the development of generalized anxiety disorder include female sex, poverty, adverse life events, chronic physical or mental illness, parental loss, insufficient emotional support as a child, or a family history of psychiatric illness. Symptoms of generalized anxiety disorder include excessive worry about several areas of life, restlessness, easy fatigability, difficulty concentrating, irritability, muscle tension, sleep disturbances, headaches, or palpitations. For a diagnosis of generalized anxiety disorder to be made, symptoms must be present more days than not for 6 months or more, cause significant impairment in daily functioning, be unrelated to substance use or a different physiologic disorder, and not be better attributed to another psychiatric disorder. Excessive worry that leads to the loss of employment represents a significant impairment in occupational functioning. Treatment of generalized anxiety disorder includes cognitive behavioral therapy and medications, such as selective serotonin reuptake inhibitors, serotonin and norepinephrine reuptake inhibitors, benzodiazepines, buspirone, and pregabalin. An abnormal cervical spine X-ray (A) would point to a physical, not psychiatric, etiology for the headaches, neck pain, shoulder pain, and poor sleep. The possibility exists to have both generalized anxiety disorder and a physical disorder, but the concurrent presence of both may make the diagnosis of either one more difficult. A Patient Health Questionnaire-9 score of 21 (C) would indicate major depressive disorder may be the cause of the patient's symptoms, not generalized anxiety disorder. On the other hand, both generalized anxiety disorder and major depressive disorder can coexist and make the diagnosis of generalized anxiety disorder a bit more difficult. Symptoms more days than not, lasting 2 to 3 weeks (D) are not present long enough to diagnose a patient with generalized anxiety disorder, where symptoms must be present for 6 months or more. A patient with anxiety symptoms that occur within 3 months of an identifiable stressor can be diagnosed with adjustment disorder.

A 26-year-old woman presents to the clinic with poor sleep. She states she has nightmares that cause her to scream and sweat several times per night. She also complains of daytime episodes of extreme anxiety, palpitations, sweating, and panic that occur after replaying mental images of a physical assault that happened to her last year. Which of the following is a common predisposing factor in the development of this patient's clinical disorder? A. Being married B. Female sex C. Higher socioeconomic status D. Older age at time of trauma

B. Female sex

A 25-year-old man presents to the clinic complaining of having trouble concentrating at work and holding down a job. He states he has never been able to sit still, even when he was a small child. He has trouble focusing, difficulty finishing projects, and often interrupts others when they are talking. He has no known chronic health conditions and takes no medications. He reports no illicit drug or alcohol use. Which of the following is required to make the most likely diagnosis? A. A total of 10 or more symptoms of hyperactivity and inattention B. Five or more symptoms of hyperactivity C. Four or more symptoms of inattention D. Symptoms that are present for at least 1 month

B. Five or more symptoms of hyperactivity A total of 10 or more symptoms of hyperactivity and inattention (A) is not necessary for a diagnosis of attention-deficit/hyperactivity disorder in an adult. A patient may have symptoms of inattention or symptoms of hyperactivity but does not necessarily need to demonstrate both for the diagnosis. Four or more symptoms of inattention (C) are too few for a diagnosis of attention-deficit/hyperactivity disorder. Five or more are needed for diagnosis in an adult, and six or more are needed for diagnosis in a child. Symptoms that are present for at least 1 month (D) do not allow sufficient time to diagnose a chronic disorder, such as attention-deficit/hyperactivity disorder. The symptoms must be present for 6 months or more. To diagnose attention-deficit/hyperactivity disorder, a patient under 17 years old must demonstrate six or more symptoms of hyperactivity or inattentiveness in the absence of physical, psychological, or situational conditions that could better account for the symptoms. A patient 17 years or older must demonstrate five or more symptoms of hyperactivity or five or more symptoms of inattentiveness for a diagnosis of attention-deficit/hyperactivity disorder. In addition to the number of symptoms, the patient must also demonstrate symptoms that are present in more than one setting, that are persistent for at least 6 months, that occur often, that presented before the age of 12 years, that impair function, and that are excessive for the developmental level of the child (if the patient is under 17 years). The World Health Organization has a self-assessment form for adults called the adult ADHD self-report scale that can be used to screen for attention-deficit/hyperactivity in adults. The self-report form includes questions regarding organizational difficulties, procrastination, fidgeting, and overactivity. Treatment for attention-deficit/hyperactivity disorder involves behavioral therapy and medications such as stimulants or alpha-2 adrenergic agonists. Pharmacologic treatment for attention-deficit/hyperactivity disorder should begin after the preschool years and may need to be lifelong, although symptoms sometimes improve with age. Pharmacologic treatment for adults involves the same type of medications as those used for children. What percentage of affected children have symptoms of attention-deficit/hyperactivity disorder that persist into adulthood? -60%

A 24-year-old woman presents to the clinic at the request of her family. Her mother reports that she has caught her daughter purging after several meals recently. The patient reports she sometimes eats two large pizzas in one sitting, purges, and then later feels guilty. She says she is highly concerned with maintaining her body weight because she is a gymnast and cannot gain weight. Her body mass index is 25 kg/m2. Which of the following is the first-line pharmacologic treatment for the suspected diagnosis? A. Bupropion B. Fluoxetine C. Olanzapine D. Sertraline

B. Fluoxetine Bupropion (A) is contraindicated in patients with current or prior eating disorders because it increases the risk of seizures in these patients. Olanzapine (C) is a second-generation antipsychotic and is the first-line medication used in patients with anorexia nervosa who are treated with pharmacologic therapy. It is not used in the treatment of bulimia nervosa. Sertraline (D) is a selective serotonin reuptake inhibitor and is considered a second-line agent for bulimia nervosa. Fluoxetine is the only agent approved by the Food and Drug Administration for bulimia nervosa and has the most studies supporting its efficacy. Common laboratory findings associated with bulimia nervosa include hypochloremia, hypokalemia, and metabolic alkalosis. The diagnosis of bulimia nervosa is made according to the criteria in the fifth edition of the Diagnostic and Statistical Manual of Mental Disorders. The recommended treatment for bulimia nervosa includes a combination of pharmacotherapy, psychotherapy, and nutritional rehabilitation. However, psychotherapy alone is more effective than pharmacotherapy alone, and cognitive behavioral therapy is the preferred psychotherapy. Pharmacotherapy alone has proven efficacy in patients who do not have access to or wish to avoid psychotherapy and nutritional rehabilitation. The preferred pharmacotherapy is fluoxetine, which is a selective serotonin reuptake inhibitor. This agent has been studied the most in clinical trials. Second-line agents include other selective serotonin reuptake inhibitors, such as sertraline. Patients who improve with pharmacotherapy should be kept on maintenance therapy for at least 6 to 12 months to prevent relapse.

A 28-year-old woman presents with a pervasive behavior pattern that includes cognitive-perceptual problems and interpersonal dysfunction. Among her symptoms is the belief she can see auras around others and she can read minds. Which of the following additional symptoms is most likely associated with the diagnosis? A. Auditory hallucinations B. Ideas of reference C. Impulsivity D. Tangential speech

B. Ideas of reference Schizotypal disorder is characterized by a long-standing and pervasive behavior pattern that includes cognitive-perceptual problems and interpersonal dysfunction associated with significant disability. It is a rare psychiatric disturbance that often co-occurs with borderline personality disorder, bipolar disorder, or panic disorder with agoraphobia. Clinical manifestations include three subgroups: cognitive-perceptual, oddness or disorganized, and interpersonal. The cognitive-perceptual symptoms are chronic distortions that include odd beliefs, unusual perceptual experiences, ideas of reference, and paranoia. Odd beliefs or magical thinking may include mind reading or thought transfer. Unusual perceptual experiences include feelings of body distortion or the thought of seeing a halo or aura around others. Ideas of reference are defined as a belief that events are occurring in direct relationship to a certain individual. For example, the belief a person on a television show is talking directly about the patient who is watching. Oddness is usually observed in these patients, who exhibit eccentric or unconventional behaviors. Interpersonal manifestations include social anxiety and an inability to develop close relationships. Auditory hallucinations (A) are often associated with schizophrenia but are not part of the diagnostic criteria for schizotypal personality disorder. Impulsivity (C) is associated with attention-deficit/hyperactivity disorder. Tangential speech (D), or derailment, is associated with schizophrenia.

A 24-year-old man presents to the clinic worried he has lung cancer. He reports his father died of lung cancer and one of his colleagues was just diagnosed with lung cancer. He has no reported symptoms other than an occasional nonproductive cough, he has no significant medical history and reports no use of any medications, supplements, or illicit substances. He does not use tobacco products. He has normal vital signs, a normal physical exam, and no known risk factors. Chart review reveals similar presentations over the past 8 months with a negative chest X-ray. Which of the following is the most likely diagnosis? A. Factitious disorder B. Illness anxiety disorder C. Somatic symptom disorder D. Specific phobia

B. Illness anxiety disorder Patients with factitious disorder (A) have external motivation behind their health concerns, which is absent in the vignette. Somatic symptom disorder (C) is similar to illness anxiety disorder, but somatic symptoms predominate and drive health fears in this condition, whereas patients with illness anxiety disorder have minimal or no somatic symptoms. Specific phobia (D) involves fear of a specified object or situation (e.g., injections, flying, animals), while the fears related to illness anxiety disorder center around having or acquiring a medical disease. Illness Anxiety Disorder Hypochondriasis Preoccupied with serious illness, despite negative exam or testing Evaluate for other medical diagnosis Management is CBT, psychotherapy, antidepressants

A 21-year-old man presents to the clinic due to recurrent nausea, vomiting, and abdominal pain. He says that, in the last 6 months, he has unintentionally lost 20 pounds. Physical exam findings are unremarkable other than mild abdominal tenderness with palpation. Urine drug screen is positive for tetrahydrocannabinol. Which of the following is most likely to confirm the diagnosis? A. Abdominal pain in the right lower quadrant B. Improvement in symptoms with hot showers C. Improvement in symptoms with ondansetron D. No improvement in symptoms with intravenous fluids

B. Improvement in symptoms with hot showers Cannabis is a drug that produces both psychoactive and antiemetic effects. Medical marijuana is now legal in many states and is sometimes used to treat nausea and vomiting due to chemotherapy. When used chronically, the overstimulation of cannabis receptors causes a paradoxical effect, resulting in recurrent nausea, vomiting, and abdominal pain. Cannabinoid hyperemesis syndrome is a type of cyclical vomiting syndrome. It occurs most commonly in young men and patients who have a history of chronic or daily marijuana use. Symptoms start in the morning and may be accompanied by bloating, diaphoresis, or weight loss. Compulsive hot showers help relieve the symptoms, and a positive history of this helps to confirm the diagnosis. Diagnosis is clinical and is made after ruling out more serious medical conditions, which can mimic the symptoms of cannabinoid hyperemesis syndrome, including pancreatitis, intestinal obstruction, acute gastroenteritis, and other disorders of the bowel. Complications of the syndrome include Mallory-Weiss tears, acute kidney injury, hypovolemia, and rhabdomyolysis. Initial treatment involves rehydration and bowel rest. Haloperidol may help to alleviate symptoms, although it should not be used in patients who are pregnant. Cannabinoid hyperemesis syndrome is ultimately cured with cessation of marijuana use. Patients may need a referral to a substance use treatment program or 12-step program to help with cessation efforts and recovery.

A 22-year-old woman presents to the clinic due to episodes of irritable mood, increased energy, and increased talkativeness that last for 2 days and are then followed by 2 days of low energy with hypersomnolence and lack of interest in activities. The episodes cause the patient distress and have been occurring for more than 2 years on most days of the year. Which of the following, if present, would indicate a diagnosis of cyclothymic disorder? A. Auditory hallucinations B. Inflated self-esteem C. No impairment in social functioning D. Suicidal ideation

B. Inflated self-esteem Cyclothymic disorder is a mood disorder characterized by mood alterations that swing between increased energy and mood and decreased energy and mood. The episodes of increased energy and irritable or expansive mood do not qualify as hypomanic or manic episodes, do not last longer than 4 days, and do not require hospitalization, but they do cause the patient distress. The episodes of low energy and depressed mood also do not meet the criteria for major depression, do not last longer than 2 weeks, and do not necessitate hospitalization. These mood disturbances should be present for at least 2 consecutive years and occur more days than not for a diagnosis of cyclothymic disorder. Symptoms of irritable or expansive mood in cyclothymia can include increased goal-directed behavior, decreased need for sleep, inflated self-esteem, talkativeness, and distractibility. Symptoms of depressed mood can include lack of interest in normal activities, increased or decreased sleep, increased or decreased appetite, poor self-esteem, guilt, and sadness. Treatment of cyclothymic disorder includes cognitive behavioral therapy and the use of low-dose mood stabilizers with the goal of achieving periods of 6 months of relative mood stability at a time. Auditory hallucinations (A) are considered a psychotic feature. Patients with expansive mood, increased energy, and increased talkativeness who also complain of auditory hallucinations meet the criteria for mania, not for cyclothymic disorder. No impairment in social functioning (C) is incorrect, one of the criteria includes symptoms cause distress or significant impairment in social or occupational functioning. Suicidal ideation (D) is a criterion for major depressive disorder and may also require hospitalization. Patients with cyclothymic disorder do not require hospitalization and do not meet the criteria for major depression.

A 53-year-old man with a history of schizophrenia was recently transitioned to a new antipsychotic drug after becoming refractory to several other antipsychotics. At his follow-up visit, he has gained 10 pounds. Which antipsychotic most commonly causes weight gain? A. Haloperidol B. Olanzapine C. Prochlorperazine D. Ziprasidone

B. Olanzapine Haloperidol (A) is a first-generation antipsychotic that can cause EPS, such as acute dystonic reactions, tardive dyskinesia, and akathisia. Many antipsychotics can also cause cardiac toxicity. Prochlorperazine (C), also a first-generation antipsychotic, causes agranulocytosis in 1-2% of patients treated. This side effect is most commonly seen between 6 and 18 weeks of treatment and is reversible with discontinuation of the drug. Ziprasidone (D), a second-generation antipsychotic, carries a high risk of QT prolongation, which can lead to life-threatening dysrhythmias. Careful consideration should be taken to not combine it with other QT-prolonging medications. Antipsychotic medications are the first-line medication in the treatment of schizophrenia. Antipsychotic medications can be broken down into two categories: first-generation, or typical, antipsychotics and second-generation, or atypical, antipsychotics. First-generation antipsychotic medications include haloperidol and chlorpromazine. Second-generation antipsychotic medications include olanzapine and risperidone. First-generation medications are used less often than second-generation medications due to the high rate of extrapyramidal symptoms (EPS) they can cause. While second-generation antipsychotics have fewer EPS, they may lead to weight gain, causing metabolic syndrome, cardiac side effects (including QTc prolongation), hyperprolactinemia, or drowsiness. Typical antipsychotics cause more side effects in general, which often cause patients to stop taking them. Side effects differ between medications and may depend on the dose. Weight gain can be a common side effect seen with the use of olanzapine. When a patient is on olanzapine, food intake, hyperglycemia, and hyperlipidemia should be monitored by obtaining fasting blood glucose and lipids every 3-6 months and obtaining a weight at each visit. Metformin has been shown to be effective in treating patients struggling with weight who are on antipsychotic medications.

An 18-year-old woman with no prior psychiatric history presents to the emergency department by ambulance after having a witnessed seizure at home. The patient's height is 5 foot 8 inches and weight is 105 pounds (BMI 16 kg/m2). Physical exam reveals lanugo on bilateral upper extremities, brittle nails, and cyanosis of the hands and feet. Laboratory workup reveals hyponatremia with hypokalemic alkalosis. In addition to seizures, which of the following complications is consistent with the suspected diagnosis? A. Hypertension B. Osteoporosis C. Polymenorrhea D. Tachycardia

B. Osteoporosis Anorexia nervosa is an eating disorder characterized by low body weight due to restriction of energy intake relative to needs with a disturbed body image and fear of gaining weight. Patients may restrict intake overall or participate in cycles of binge eating and purging. Medical complications account for approximately half of all deaths in anorexia nervosa, which has one of the highest mortality rates of any mental illness. The general complications of anorexia nervosa are a direct result of weight loss and malnutrition. Starvation induces protein and fat catabolism, which leads to loss of cellular volume and atrophy of the heart, liver, brain, intestines, kidneys, and muscles. Common complications of the cardiovascular system include decreased cardiac mass, reduced cardiac chamber volumes, valvular disorders, bradycardia, hypotension, and QT prolongation. Gynecological complications include infertility and secondary amenorrhea due to anovulation, which often resolve after proper weight gain. Patients with anorexia nervosa can experience profound bone loss due to malnutrition causing increased resorption and decreased formation of bone and are therefore at increased risk of developing osteoporosis. Gastrointestinal complications, including gastroparesis and constipation, are commonly seen. However, patients may also experience diarrhea. Patients may also experience reduced glomerular filtration rate and problems concentrating urine, leading to electrolyte abnormalities, dehydration, and seizures. Neurological disorders, such as Wernicke encephalopathy, Korsakoff syndrome, and brain atrophy, may also be seen and can also contribute to seizures. Cytopenias, including anemia, are often seen. Pulmonary function is often decreased due to wasting of respiratory muscles, resulting in dyspnea and reduced aerobic capacity. Treatment of anorexia nervosa includes refeeding, which may require hospitalization and supervision in severe cases to prevent refeeding syndrome, cognitive behavioral therapy, and treatment of complications as indicated. Psychiatric medications, including selective serotonin reuptake inhibitors (SSRIs) and antipsychotics, play a limited role but may be indicated to treat comorbid illness.

A 26-year-old woman presents to the clinic with excessive worrying for the past 7 months. She reports poor sleep, fatigue, difficulty relaxing, and frequent tension-type headaches. You decide to start her on a medication from the same class as paroxetine. Which of the following is an adverse side effect of this class of medications? A. Dependence B. Sexual dysfunction C. Shortening of the QT interval D. Weight loss

B. Sexual dysfunction Generalized anxiety disorder is a psychiatric condition marked by excessive and persistent worry that is difficult to control and results in significant distress and impairment of everyday function. The etiology has both genetic and environmental components. Risk factors for generalized anxiety disorder include female sex, low income, recent adverse life events, chronic illness, parental loss, and a history of psychiatric disorder in parents. The excessive worry spans over many aspects of life. Furthermore, somatic symptoms often accompany the worrying. Patients often report poor sleep, fatigue, difficulty relaxing, headaches, and pain in the neck, shoulders, or back. The diagnosis is made according to the criteria in the fifth edition of the Diagnostic and Statistical Manual of Mental Disorders and is largely based on the classic symptoms. Symptoms must be present on most days for at least 6 months to meet the diagnostic criteria. The treatment options for generalized anxiety disorder include psychotherapy, pharmacotherapy, or both. Serotonergic reuptake inhibitors, which include selective serotonin reuptake inhibitors (sertraline and escitalopram) and serotonin and norepinephrine reuptake inhibitors (duloxetine and venlafaxine), are considered the first-line pharmacologic treatment of generalized anxiety disorder. These medications have similar efficacy, and the selection of a particular agent is based mainly on the slight variances in their side effect profiles. Common side effects of selective serotonin reuptake inhibitors include sexual dysfunction, gastrointestinal side effects (nausea and diarrhea), insomnia, somnolence, and weight gain. Common side effects of serotonin and norepinephrine reuptake inhibitors include nausea, dizziness, insomnia, sedation, and constipation. The clinical response should be evaluated after 4-6 weeks of treatment since these medications take several weeks to reach their full efficacy. Patients with no response should try another agent from either the selective serotonin reuptake inhibitor or serotonin and norepinephrine reuptake inhibitor class. Patients with a partial response can be treated with buspirone or pregabalin, in addition to the serotonergic agent, to augment the partial response. Common side effects of buspirone include insomnia, agitation, and nausea, while side effects of pregabalin can include sedation and dizziness. Patients who experience a good response should continue the serotonergic medication for at least 12 months to prevent relapse or recurrence. Benzodiazepines, such as lorazepam or clonazepam, are sometimes used for acute management of anxiety and worry during the first few weeks after a serotonergic agent is initiated. While benzodiazepines have efficacy in reducing symptoms of generalized anxiety disorder, there is potential for dependence and tolerance to develop. Dependence (A) is incorrect. However, stopping selective serotonin reuptake inhibitors abruptly or rapidly tapering can cause discontinuation or withdrawal symptoms, such as headache, dizziness, fatigue, and nausea. Selective serotonin reuptake inhibitors can prolong, not shorten, the QT interval (C). Weight gain, not weight loss (D), is a common side effect.

What is the only antidepressant that is not known to aggravate restless legs syndrome?

Bupropion

A 35-year-old man presents to the clinic for an annual wellness exam. You ask him about tobacco use, and he reports that he smokes cigarettes daily. According to the five As approach to helping patients with tobacco cessation, which of the following is the third step? A. Advise quitting B. Arrange follow-up C. Assess readiness to quit D. Assist smokers who are ready to quit

C. Assess readiness to quit The U.S. Preventive Services Task Force recommends that clinicians use the five As approach to help patients stop using tobacco. In sequential order, the five As are ask, advise, assess, assist, and arrange. The first step is for the clinician to ask the patient about tobacco use. Patients who report using tobacco should be advised to quit using tobacco, which is the second step. The third step is to assess the patient's readiness to quit, as smokers vary in their readiness to change tobacco use. The Stages of Change is a model that discusses five stages patients may go through on the continuum between smoking and abstinence: precontemplation (not ready to quit), contemplation (considering a quit attempt), preparation (actively planning a quit attempt), action (actively involved in a quit attempt), and maintenance (achieved smoking cessation). The fourth step is to assist smokers who are ready to quit, which is defined as anyone who is at least at the contemplation stage. It is important for the clinician to understand what methods the patient has previously used to quit smoking and whether the patient has had any success with these methods. Patients who want to quit using tobacco should set a quit date. The clinician should address barriers to quitting (e.g., triggers to smoke) and discuss the possibility of nicotine withdrawal symptoms. The symptoms of nicotine withdrawal peak in the first 3 days of smoking cessation and then gradually subside over the next 3-4 weeks. The symptoms of nicotine withdrawal include increased appetite, weight gain, changes in mood (dysphoria or depression), insomnia, irritability, anxiety, and restlessness. These symptoms can be relieved by pharmacologic treatment, including nicotine replacement, varenicline, or bupropion. The fifth step of the five As approach is to arrange follow-up. The recommended time for follow-up is 1-2 weeks after the patient's quit date. This timing of follow-up allows the clinician to monitor smoking cessation therapy, including adverse effects of pharmacotherapy, and address any questions the patient may have. Advise quitting (A) is the second step, assist smokers who are ready to quit (D) is the fourth step, and arrange follow-up (B) is the fifth step of the five As approach to helping patients with tobacco cessation.

A 7-year-old boy presents to the clinic with his mother who is concerned because he has been craving ice for the past 2 months. She reports that he consumes about 10 cups of ice per day. Which of the following nutritional deficiencies is the history most concerning for? A. Cyanocobalamin deficiency B. Folic acid deficiency C. Iron deficiency D. Magnesium deficiency

C. Iron deficiency Pica is a psychological disorder defined by an intense craving for nonfood items. The onset of symptoms typically begins during childhood but may also begin during adolescence or adulthood. The diagnosis is based on clinical criteria as defined in the fifth edition of the Diagnostic and Statistical Manual of Mental Disorders. The clinical criteria include repeated eating of nonfood substances that are not nutritional for at least 1 month. Further, the eating behavior must be inappropriate for the patient's expected developmental stage. Pagophagia is a specific type of pica defined as a craving for ice. Pagophagia is specific to iron deficiency, although the mechanism behind this association is unclear. Pagophagia improves rapidly with iron supplementation.

A 22-year-old man presents to his therapist to discuss his fear of spiders. He reports that every night before he goes to sleep he takes off every pillowcase and sheet to ensure there are no spiders in his bed. He also avoids outdoor activities in which he may encounter a spider, including walking around the neighborhood and going in his backyard. Which of the following is the recommended first-line treatment for the suspected condition? A. Alprazolam B. Cognitive behavioral therapy that includes imaginal exposure C. Cognitive behavioral therapy that includes real-world exposure D. Sertraline

C. Cognitive behavioral therapy that includes real-world exposure Alprazolam (A) is a benzodiazepine. Benzodiazepines can be used as needed as second-line therapy, short term, for patients who do not have access to cognitive behavioral therapy with exposure, cannot tolerate cognitive behavioral therapy with exposure, or who prefer medications. Cognitive behavioral therapy that includes imaginal exposure (B) is acceptable, but real-world exposure is preferred unless it is too infrequent or too expensive to recreate. Sertraline (D) is a selective serotonin reuptake inhibitor. Selective serotonin reuptake inhibitors are not typically used to treat patients with specific phobias because there is minimal evidence to support their use and a potential for adverse effects, such as sexual dysfunction. Specific phobias are a type of anxiety disorder in which patients experience clinically significant anxiety or fear related to anticipation of or exposure to situations or objects. The anxiety frequently leads to avoidant behaviors in which individuals attempt to reduce the likelihood of being exposed to the feared situation or object. Specific phobias begin most often during childhood or adolescence. The pathogenesis has both genetic and environmental components. The main categories of specific phobias are animal, natural environment (heights, storms, or water), blood-injection-injury, situational (airplanes, elevators, or enclosed spaces), and other. The severity of functional impairment varies according to the degree of the fear and the situation or object of fear. For example, fear of snakes may have minimal impact on an individual's life, whereas fear of injections may lead to an individual avoiding necessary medical procedures. The recommended treatment of specific phobias is cognitive behavioral therapy that includes real-world exposure. Benzodiazepines can be used as a second-line treatment when cognitive behavioral therapy with exposure is unavailable or cannot be tolerated, or in patients who prefer medications on a short-term basis.

A 59-year-old man with a past medical history of obesity, hypertension, generalized anxiety disorder, and type 2 diabetes mellitus presents to the primary care clinic for his annual wellness exam. Medications include lisinopril 40 mg once daily, sertraline 50 mg once daily, and metformin 1,000 mg twice daily. His blood pressure today is 150/90 mm Hg, and his A1C is 9.2%. During his visit, the patient mentions concerns about worsening difficulty achieving and maintaining an erection for the past 3 months. He is upset about these symptoms and is requesting medication. What is the most appropriate next step in management of the patient's suspected diagnosis? A. Begin cognitive behavioral therapy B. Begin sildenafil as needed C. Decrease dose of sertraline D. Use of a vacuum-assisted device

C. Decrease dose of sertraline Beginning cognitive behavioral therapy (A) may be helpful, especially in the instance of comorbid psychiatric conditions, but it is not considered a first-line therapy for ED. Beginning sildenafil as needed (B) would be most appropriately done after lifestyle changes and medication adjustments have been made to better control comorbid conditions and sertraline is decreased in dosage or changed to another medication. The patient will likely see improvement or resolution in symptoms after these changes are made. Use of a vacuum-assisted device (D) may be necessary if first-line management, including watchful waiting and medication adjustments, fails.

A 42-year-old man presents to marital counseling with his wife who he states is being unfaithful. He believes she is unfaithful because she typically arrives home late from work about twice a month and has male coworkers. His wife has attempted to reassure him by sharing her location with him and having several other coworkers vouch that she is working the nights she comes home late. Nonetheless, he remains unconvinced. This has led to conflict over the past 6 months. Which of the following is the most likely diagnosis? A. Bipolar disorder type 1 B. Brief psychotic disorder C. Delusional disorder D. Schizophrenia

C. Delusional disorder Bipolar disorder type 1 (A) is a condition defined by the presence of at least one manic episode. Manic episodes are defined by a persistently elevated or irritable mood that lasts for at least a week and causes an increase in goal-directed behavior. Patients who are experiencing a manic episode may have delusions, but patients with delusional disorder must experience delusions in the absence of a manic episode. The patient in the vignette does not meet the criteria for a manic episode. Brief psychotic disorder (B) is marked by hallucinations or delusions that last more than a day but less than a month. The patient described in the vignette has a symptom duration of at least 6 months. Schizophrenia (D) is another psychiatric condition that may cause delusions. However, patients with delusional disorder do not exhibit other important features of schizophrenia, such as hallucinations (unless they are associated with the delusion), disorganized thoughts or behavior, and negative symptoms. The patient in the vignette does not have these other symptoms to suggest schizophrenia. Delusional Disorder One or more delusions for at least 1 month Criterion A for schizophrenia never met Apart from the impact or ramifications of the delusion(s), functioning not markedly impaired and behavior not obviously odd or bizarre Erotomanic: belief that another person is in love with the individual Grandiose: conviction of having some great talent or insight Jealous: delusion that partner is unfaithful Persecutory: belief that being conspired against, spied on, poisoned, etc. Somatic: delusion about bodily functions or sensations With bizarre content: delusions are clearly implausible, not understandable, and not derived from ordinary life experiences (e.g., belief that a stranger removed one's internal organs and replaced with another's organs without leaving a scar)

A 19-year-old woman presents to the primary care clinic with decreased pleasure in usually pleasurable activities for the past 3 months. She also reports hypersomnia, decreased appetite, difficulty concentrating, and fatigue. Which of the following is more prevalent for the suspected diagnosis? A. Asian ancestry B. Cigarette smoking C. Divorced marital status D. High socioeconomic status

C. Divorced marital status Major depressive disorder is a condition marked by depressed mood or dysphoria that lasts at least 2 weeks and is accompanied by at least five of the following features: anhedonia, changes in appetite, changes in weight, sleep disturbances (insomnia or hypersomnia), fatigue, neurocognitive dysfunction, feelings of guilt, and suicidal ideation. Depression is most common in young adults and is more common in women than men. Risk factors for depression include a family history of first-degree relatives having depression; low socioeconomic status; marital status of divorced, separated, or widowed; lack of interpersonal relationships; recent difficult life events; early childhood trauma (e.g., childhood physical abuse, childhood emotional abuse, or neglect); and postpartum status. The diagnostic criteria for major depressive disorder are listed in the fifth edition of the Diagnostic and Statistical Manual of Mental Disorders and require that the symptoms last at least 2 weeks and cause clinically significant impairment. The first-line treatment for major depressive disorder is antidepressants. Selective serotonin reuptake inhibitors, such as sertraline or escitalopram, are typically the antidepressant of choice due to their safety profile. Individuals of American Indian descent have a higher prevalence of depression compared to other ethnicities in the United States, and individuals of Asian ancestry (A) have the lowest prevalence. Cigarette smoking (B) is not a known risk factor for major depressive disorder. High socioeconomic status (D) is incorrect because the prevalence of major depressive disorder is higher in individuals with low socioeconomic status.

A 23-year-old woman presents to the clinic with decreased pleasure in usually enjoyable activities for the past month. She also reports hypersomnia, weight gain, decreased concentration, and excessive guilt. Which of the following is the most likely diagnosis? A. Bipolar disorder type 2 B. Cyclothymic disorder C. Major depressive disorder D. Persistent depressive disorder

C. Major depressive disorder Bipolar disorder type 2 (A) is diagnosed in patients who meet the criteria for a major depressive episode and a hypomanic episode. Hypomania is defined by an abnormal and persistently elevated or irritable mood, an increase in energy or goal-directed activity that lasts at least 4 days, and at least four of the symptoms that characterize mania. Hypomanic episodes are less severe and have a shorter duration than manic episodes. Cyclothymic disorder (B) is defined by recurrent episodes of hypomanic or depressive symptoms that do not meet the criteria for bipolar disorder type 2 or major depressive disorder, respectively. The diagnostic criteria for cyclothymic disorder require that symptoms be present at least half the time for a period of 2 years. Persistent depressive disorder (D) is defined by depressed mood for at least 2 consecutive years with at least two of the following symptoms: decreased or increased appetite, insomnia or hypersomnia, low energy or fatigue, low self-esteem, impaired concentration or decision making, and hopelessness.

A 48-year-old man presents to the clinic complaining of fatigue. He reports drinking 6 to 10 beers per day for the past 15 years. He currently takes lisinopril for hypertension. Physical exam reveals rhinophyma, facial erythema, hepatomegaly, an abdominal fluid wave, and external hemorrhoids. Which of the following laboratory abnormalities would be most likely in this patient? A. Alanine aminotransferase to aspartate aminotransferase ratio of 4:1 B. Gamma-glutamyltransferase 20 IU/L C. Mean corpuscular volume 105 fL D. Serum albumin 5.7 g/dL

C. Mean corpuscular volume 105 fL Alcohol use disorder can be diagnosed when a patient exhibits recurrent drinking leading to neglecting obligations, recurrent drinking in high-risk situations, drinking that leads to interpersonal problems, evidence of tolerance to alcohol, evidence of alcohol withdrawal, alcohol craving, desire to stop drinking with unsuccessful quit attempts, and a great deal of time spent surrounding alcohol-related pursuits. Patients at increased risk for alcohol use disorder are those under age 24, and men. Health problems attributable to alcohol use disorder include hypertension, anxiety, depression, gastrointestinal reflux, sleep disturbance, liver disease, bone marrow suppression, macrocytosis (mean corpuscular volume over 96 fL), cardiomyopathy, neuropathy, and trauma due to falls. Facial erythema, rhinophyma, superficial spider hemangiomas, varices, and hemorrhoids can also occur with chronic alcohol use. Treatment of alcohol use disorder depends on disease severity. Patients with mild disease benefit from psychosocial intervention alone, while those with moderate-to-severe disease often require an initial period of medically managed detoxification with benzodiazepines and antihypertensives. After initial withdrawal, patients with moderate-to-severe disease may benefit from psychosocial interventions plus naltrexone. Liver function tests are often abnormal in patients with alcohol use disorder. In general, a ratio of aspartate aminotransferase to alanine aminotransferase of 2:1 is indicative of alcohol use disorder, not an alanine aminotransferase to aspartate aminotransferase ratio of 4:1 (A). Gamma-glutamyltransferase 20 IU/L (B) is within normal limits. Patients with alcohol use disorder will often have gamma-glutamyltransferase levels well above the upper limits of normal (over 30 IU/L). Serum albumin 5.7 g/dL (D) is slightly above the normal range. Patients with alcohol use disorder are often undernourished and may have serum albumin levels under the lower limits of normal (3.4 g/dL).

A 7-year-old girl presents to the pediatrician multiple times for hematuria until it is discovered that her mother is contaminating the urine samples with her own blood. Which of the following is the suspected diagnosis? A. Conversion disorder B. Factitious disorder C. Medical child abuse D. Somatic symptom disorder

C. Medical child abuse Conversion disorder (A) is a condition marked by neurologic symptoms that are not consistent with known neurologic or general medical conditions. Factitious disorder (B) occurs when patients intentionally fabricate symptoms, with the goal being to play the role of the patient. Medical child abuse is when the caregiver fabricates symptoms in a child. Somatic symptom disorder (D) is a condition marked by the presence of longstanding somatic symptoms, such as pain or fatigue, that cause significant and excessive worrying and time devotion. The symptoms may or may not be explained by a general medical condition. Medical child abuse, which was formerly known as Munchausen syndrome by proxy, is diagnosed when a child receives unnecessary medical care that is harmful or potentially harmful because of a caregiver's actions. The caregiver may exaggerate symptoms, lie about symptoms, fabricate exam findings, or intentionally produce illness in a child. The clinical manifestations of medical child abuse vary significantly and include almost any presentation. However, it is common for the symptoms to be unwitnessed. Symptoms may include vomiting, diarrhea, hematuria, seizures, apneic episodes, and allergic reactions. The presentation of medical child abuse can be divided into mild, moderate, or severe based on the consequences for the child. In mild cases of medical child abuse, the caregiver may seek unnecessary diagnostic tests or treatment, but significant disruption in the child's life does not occur. Moderate cases of medical child abuse are marked by significant disruption in the child's life without the child's life being threatened. Examples include the caregiver causing rashes by painting on the skin or insisting on a wheelchair for a child who does not have a medical need for one. Severe cases of medical child abuse are potentially life-threatening either from the induction of illness or the medical care received. One example is a child having multiple episodes of polymicrobial sepsis due to a parent injecting feces in a central venous catheter. Clinicians must have a low index of suspicion for medical child abuse to make the diagnosis. In cases in which medical child abuse is suspected, particularly moderate or severe abuse, the clinician should consult with a multidisciplinary child abuse team.

A 19-year-old girl presents to the clinic for an annual physical. She states she runs at least 45 miles per week to maintain her figure. The patient has no significant medical history and has been amenorrheic for 6 months. Physical examination reveals a thin patient with peripheral edema and fine, downy hair on her face and arms. Vital signs include body mass index of 16.52 kg/m2, HR of 52 bpm, RR of 17 breaths per minute, BP of 98/52 mm Hg, T of 97.2°F, and SpO2 of 97% on room air. Which of the following is the best diagnosis? A. Extreme anorexia nervosa B. Mild anorexia nervosa C. Moderate anorexia nervosa D. Severe anorexia nervosa

C. Moderate anorexia nervosa Extreme anorexia nervosa (A) is diagnosed as a body mass index < 15 kg/m2, while mild anorexia nervosa (B) is characterized by a body mass index > 17 kg/m2. Patients with severe anorexia nervosa (D) have a body mass index between 15 and 15.99 kg/m2. Anorexia nervosa is an eating disorder that commonly presents during the adolescent and early adult years of life and most frequently affects women. This condition is characterized by self-imposed starvation due to a distorted perception of body weight and shape. Core diagnostic features of this disorder include a persistent restriction in energy intake that results in low body weight, an intense fear of gaining weight, and a distorted view of body image. Patients may engage in binge eating and purging practices or practice restriction via fasting, excessive exercise, or extreme dieting. The severity of anorexia nervosa is classified by body mass index, with a value ≥ 17 kg/m2 indicating mild disease, values between 16 and 16.99 kg/m2 representing moderate disease, and a body mass index of 15-15.99 kg/m2 defined as severe disease. Extreme anorexia nervosa is diagnosed in patients with a body mass index < 15 kg/m2. Anorexia commonly manifests with signs of starvation and malnutrition, including emaciation, significant hypotension, bradycardia, hypothermia, lanugo, peripheral edema, and amenorrhea or delayed menarche in prepubescent girls. Patients who practice self-induced vomiting may also have salivary gland hypertrophy, dental enamel erosion, and calluses or scars on the back of the hand and fingers (Russell sign). Patients may report exertional fatigue, dizziness, weakness, cold intolerance, palpitations, constipation, abdominal pain and bloating, and swelling of the feet. Laboratory results outside the reference range that can be due to chronic starvation include leukopenia, hypokalemia, hypochloremia, low thyroid hormone levels, and elevated liver enzymes, cortisol, and serum bicarbonate. Additionally, women often have low estrogen concentrations. Patients with anorexia nervosa who demonstrate medical instability (e.g., severe bradycardia or hypotension, cardiac dysrhythmia, body mass index < 15 kg/m2) require inpatient treatment with assessment and appropriate management of medical complications. Standard treatment procedures for patients with anorexia who do not require medical stabilization include nutritional rehabilitation and psychotherapy. Pharmacotherapy using antipsychotics, specifically olanzapine, can be added to an established regimen to promote weight gain. Distorted thoughts of body image or food are generally not amenable to pharmacotherapeutic approaches. Patients with comorbid depression or anxiety may benefit from selective serotonin reuptake inhibitor therapy (e.g., sertraline, fluoxetine), but tricyclic antidepressants (e.g., amitriptyline, nortriptyline) should be avoided in patients with anorexia, and bupropion is contraindicated in patients with eating disorders.

A 25-year-old man presents to the emergency department with altered mental status. Vital signs are significant for a temperature of 102.5°F, heart rate of 135 bpm, and blood pressure of 195/122 mm Hg. You examine him and find a stuporous man with profuse diaphoresis, rigidity in his extremities through all ranges of movement. You discuss his presentation with his mother, who notes he recently started taking risperidone for an unknown psychiatric condition. CT scan of his head is negative, and lumbar puncture reveals cerebrospinal fluid of 5 white blood cells/µL. Which of the following diagnoses is suspected? A. Bacterial meningitis B. Malignant hyperthermia C. Neuroleptic malignant syndrome D. Serotonin syndrome

C. Neuroleptic malignant syndrome Bacterial meningitis (A) presents with fever, headache, neck stiffness, and altered mental status. The diagnosis is supported by a cerebrospinal fluid white blood cell count of more than 1,000 white blood cells/µL. The diagnosis is confirmed by bacterial growth on a culture of cerebrospinal fluid. Malignant hyperthermia (B) is a rare genetic disorder that presents with similar clinical features as neuroleptic malignant syndrome. It is distinguished by the clinical setting since malignant hyperthermia occurs with use of potent halogenated anesthetic agents and succinylcholine. Serotonin syndrome (D) occurs in individuals who are taking serotonergic agents, such as selective serotonin reuptake inhibitors. It also causes similar symptoms to neuroleptic malignant syndrome. However, nausea, vomiting, and diarrhea are more common in serotonin syndrome. Typically patients with serotonin syndrome have hyperreflexia while neuroleptic malignant syndrome rarely has hyperreflexia. Neuroleptic malignant syndrome is a life-threatening condition associated with the use of dopamine antagonist agents or withdrawal from dopamine agonists. Common dopamine antagonists are antipsychotic medications and the antiemetics metoclopramide, prochlorperazine, and promethazine. The hallmark clinical tetrad of neuroleptic malignant syndrome is mental status changes, rigidity, fever, and dysautonomia. Dysautonomia consists of tachycardia, labile or high blood pressure, tachypnea, and diaphoresis. Lead-pipe rigidity is a classic exam finding and is defined by rigidity that produces smooth and steady resistance to passive movement of the limbs. Neuroleptic malignant syndrome develops most often within 2 weeks of starting a new antidopaminergic medication, but it can also develop after one dose or in a patient who has been on a stable dosing regimen for years. Common laboratory findings include elevated creatine kinase and leukocytosis. The diagnosis is supported by the presence of at least two of the four findings in the tetrad in a patient who takes a medication known to be associated with neuroleptic malignant syndrome. Neuroimaging and lumbar puncture for cerebrospinal fluid analysis must be obtained to rule out central nervous system infection and structural brain disease. Patients with neuroleptic malignant syndrome should be treated in the intensive care unit. Stopping the causative agent is the most important intervention. Otherwise, supportive care is the mainstay of treatment. Supportive care includes maintaining a euvolemic state via intravenous fluids, treating hyperthermia with cooling blankets, lowering blood pressure when it is markedly elevated, preventing venous thromboembolism with anticoagulation, and treating agitation with benzodiazepines. Some medications (including dantrolene, bromocriptine, and amantadine) are possibly effective in moderate or severe cases.

A 20-year-old woman presents to the office reluctantly with her roommates, who express concerns about recent changes in the patient's behavior. The patient has recently had a hard time leaving the apartment for fear she will leave her hair straightener plugged in. She has to check multiple times before leaving and is over an hour late to her first class every day. The patient is quiet and reluctant to speak. On physical examination, you notice small patches of hair missing from her scalp. What is the most likely diagnosis? A. Generalized anxiety disorder B. Major depressive disorder C. Obsessive-compulsive disorder D. Obsessive-compulsive personality disorder

C. Obsessive-compulsive disorder Generalized anxiety disorder (A) is a mental disorder characterized by persistent and excessive worrying that lasts at least 6 months. The worrying is hard to control and causes significant distress to the patient involving real-life concerns. Major depressive disorder (B) is characterized by five or more of the following for at least 2 weeks: depressed mood, anhedonia, changes in appetite or weight, changes in sleep, feelings of worthlessness or guilt, decreased ability to concentrate, and recurrent thoughts of death. At least one of the symptoms must be either depressed mood or anhedonia to make a diagnosis. Patients may have ruminative thoughts, but they are not related to obsessions or compulsions. Obsessive-compulsive personality disorder (D) does not involve obsessions but is characterized by a pattern of perfectionism and control leading to repetitive behaviors. Obsessive-compulsive disorder (OCD) is a mental health disorder in which the patient experiences recurring, distressing thoughts (obsessions) or repetitive mental or behavioral actions (compulsions). OCD initially presents in childhood or adolescence and persists throughout the patient's lifetime. Patients may experience both or only one symptom. Obsessions are distressing, disturbing, and unwanted by the patient. They may present as recurrent thoughts such as the fear of germ exposure or contamination, images of violent scenes, or urges such as the urge to harm someone. Patients develop compulsions as a way to suppress the obsession or prevent it from coming true. Compulsions can be a physical or mental action, such as repetitively checking, washing, or counting. The obsessions and compulsions a person suffers from are specific to that person, but there are several themes that they usually follow, including cleaning, symmetry (e.g., repeating, ordering, counting), forbidden thoughts (e.g., aggressive or sexual thoughts), and harm. To make a diagnosis according to the DSM-5 criteria, a patient may have obsessions, compulsions, or both. The obsessions or compulsions last > 1 hour per day or cause significant distress to the patient, are not due to other substance use or medication, and are not explained by another mental disorder.

A 55-year-old man who takes medications for hypertension, hyperlipidemia, and major depressive disorder presents to the clinic to discuss sexual dysfunction. He also mentions he has been experiencing nausea recently as well as sedation. Which of the following medications is likely responsible for his symptoms? A. Bupropion B. Mirtazapine C. Paroxetine D. Venlafaxine

C. Paroxetine Selective serotonin reuptake inhibitors (SSRIs) are the most common class of medications used to treat major depressive disorder. Gastrointestinal (GI) upset and sleep disturbances are common side effects of these medications. Paroxetine is a potent SSRI, and its mechanism of action is to inhibit the uptake of serotonin by the central nervous system, thus increasing available serotonin levels. Paroxetine is known to cause sexual dysfunction, nausea, sedation, and sometimes weight gain. Sexual dysfunction, GI upset, and sleep disturbances can be common side effects among SSRI medications. Bupropion (A) is a norepinephrine and dopamine reuptake inhibitor used to treat major depressive disorder. Bupropion is known to cause gastrointestinal side effects such as nausea, but it is not known to cause sexual side effects. Mirtazapine (B) is a tetracyclic antidepressant medication that can cause sedation but does not cause sexual side effects. Mirtazapine also causes weight gain, hypercholesterolemia, and constipation. Venlafaxine (D) is a serotonin and norepinephrine reuptake inhibitor, and common side effects include gastrointestinal upset, weight loss, and sleep disturbance. Sexual dysfunction is not a common side effect of venlafaxine.

A 45-year-old man with a history of heroin dependence presents to the clinic requesting medical treatment for his addiction. He has tried both cold turkey and 12-step programs without success. He is specifically interested in trying methadone. Which of the following is a potentially life-threatening adverse effect of methadone? A. Atrial fibrillation B. Hypertensive emergency C. QT prolongation D. ST depression

C. QT prolongation Atrial fibrillation (A) is not associated with methadone use. Methadone causes respiratory and central nervous system depression. It does not cause an increase in blood pressure, thus hypertensive emergency (B) is not a side effect that requires monitoring. Other cardiac side effects may occur with the use of methadone. ST elevation, angina pectoris, bradycardia, ventricular tachycardia, myocardial infarction, or cardiac arrest may occur. ST depression (D) is not a finding seen on ECG in patients taking methadone. Opioids in the medical setting are used for pain relief. They also have the potential to cause euphoria. This effect is what leads to opioid misuse and the development of opioid use disorder. Opioid use disorder can occur with both pharmaceutical opioids, such as oxycodone or morphine, or illegal opioids, such as heroin. It is a chronic disorder, typically with multiple episodes of relapse and a high rate of both morbidity and mortality. Opioid use disorder is diagnosed using the fifth edition of the Diagnostic and Statistical Manual of Mental Disorders (DSM-5) criteria, including taking opioids in larger amounts or over a longer period of time than intended; unsuccessful attempts to cut down or quit; cravings; interference with obligations at home, school, or work; use in situations that are physically hazardous; development of tolerance; and the presence of withdrawal symptoms. Medication-assisted treatment with an opioid agonist or antagonist, together with psychosocial interventions is considered first-line treatment for individuals with opioid use disorder. Methadone is a long-acting opioid agonist, which acts on the mu-opioid receptors, resulting in the prevention of withdrawal symptoms, a decrease in cravings, and a reduction in the euphoric effects of other opioids if used while taking methadone. When prescribed for opioid use disorder, methadone is regulated and must be dispensed at a methadone clinic. There are a number of side effects and many drug interactions associated with methadone. Constipation, sweating, drowsiness, and peripheral edema are common side effects, but the cardiac side effects are the most dangerous, and patients on methadone need to be monitored closely to prevent them from occurring. Methadone use has been associated with QT prolongation, which can lead to cardiac dysrhythmias. There are a number of risk factors that increase the likelihood of developing QT prolongation while taking methadone, including age > 65 years, female sex, liver and kidney disease, hemodialysis, anorexia nervosa, and cardiovascular disease. Prior to starting a methadone program, patients should be advised of the risk of dysrhythmia; screened for a history of structural heart disease, dysrhythmia, or syncope; and assessed for other risk factors associated with QT prolongation.

A 21-year-old woman presents with concerns for fatigue and abdominal discomfort. She reports her symptoms began 3 months ago when she started dieting. She feels she has no control over the amount of food she eats at times, and in an effort to compensate for the amount of food she eats, she resorts to vomiting after meals or using laxatives. Which of the following may be found on physical exam when considering the most likely diagnosis? A. Hypertension B. Sinus bradycardia C. Tooth enamel erosion D. Unilateral parotitis

C. Tooth enamel erosion Hypertension (A) is not consistent with bulimia nervosa, as typically patients with this disorder are volume depleted and thus exhibit hypotension. Sinus bradycardia (B) is not as likely as sinus tachycardia for the same reason. Volume depletion causes an increased heart rate. Unilateral parotitis (D) is not associated with bulimia nervosa. However, hypertrophy of the parotid gland bilaterally is more common and is secondary to self-induced vomiting. Bulimia nervosa is characterized by episodes of binge eating followed by inappropriate compensatory behavior that occurs at least once per week for at least 3 months. This behavior is secondary to an unhealthy perception of body image. During the periods of binge eating, individuals often feel they have no control over the amount of food they are consuming. Caloric restriction is a type of compensatory behavior that typically propagates the cycle of binge eating and vomiting. Compensatory behavior is most often demonstrated by self-induced vomiting but can include the inappropriate use of laxatives or emetics. Medical symptoms of bulimia nervosa commonly include lethargy, irregular menses, abdominal pain, or constipation. Physical exam findings may include tachycardia, hypotension, xerosis, sialadenosis, and tooth enamel erosion. The tooth enamel erosion is caused by frequent vomiting and contact with gastric acid. Laboratory studies should include a complete metabolic panel, complete blood count with differential, and urinalysis.

Which childhood condition often precedes antisocial personality disorder?

Conduct disorder

A 20-year-old woman is seeing a therapist for a condition marked by repetitive excessive food intake followed by self-induced vomiting. These behaviors have been happening several times per week for 6 months, and the patient reports an intense fear of gaining weight. Her body mass index is 29.2 kg/m2. Which of the following is the most likely diagnosis? A. Anorexia nervosa B. Binge eating disorder C. Borderline personality disorder D. Bulimia nervosa

D. Bulimia nervosa Anorexia nervosa (A) is an eating disorder marked by restricted caloric intake leading to an abnormally low body mass index, intense fear of gaining weight, and a distorted perception of body weight and shape. The body mass index is the main distinguishing factor between anorexia nervosa and bulimia nervosa, as patients with bulimia nervosa have a normal or high body mass index. Binge eating disorder (B) is an eating disorder marked by binge eating episodes. It is distinguished from bulimia nervosa by the lack of compensatory behaviors that are seen in bulimia nervosa. Borderline personality disorder (C) is a cluster B personality disorder that is marked by emotional impulsivity. The other core features of borderline personality disorder are unstable interpersonal relationships and unstable self-image, suicidal behavior, and inappropriate intense anger.

A 26-year-old woman presents for psychotherapy. You notice she seems to use physical appearance to draw attention to herself and considers relationships to be more intimate than they actually are. Which of the following additional findings is most consistent with the suspected personality disorder? A. Cognitive and perceptual disturbances B. Disregard for the rights of others C. Distrust and suspiciousness of others D. Excessive emotionality and attention-seeking behavior

D. Excessive emotionality and attention-seeking behavior Histrionic personality disorder is a cluster B personality disorder marked by excessive emotionality and attention-seeking behavior. It is more common in women. Histrionic personality disorder is diagnosed according to the criteria in the fifth edition of the Diagnostic and Statistical Manual of Mental Disorders. The criteria include a pervasive pattern of excessive emotionality and attention-seeking that is present in a variety of contexts by early adulthood. Furthermore, at least five of the following symptoms must be present: discomfort with not being the center of attention, seductive behavior, quickly shifting and superficial emotional expression, use of physical appearance to draw attention to oneself or evade unwanted responsibilities, impressionistic style of speech that lacks in detail, exaggerated expression of emotion, influenced easily by others, and tendency to consider relationships more intimate than they are. The preferred treatment is psychotherapy, specifically cognitive behavioral therapy or group therapy. Cognitive and perceptual disturbances (A) are features that mark schizotypal personality disorder. Disregard for the rights of others (B) is a feature that marks antisocial personality disorder. Distrust and suspiciousness of others (C) are features that mark paranoid personality disorder.

A 25-year-old man presents to his therapist to discuss his difficulty maintaining romantic relationships. He reports that he is sexually aroused by his partners' toes, which has ended several of his past relationships. Which of the following is another common focus of sexual arousal in the suspected condition? A. Bracelets B. Ears C. Eyes D. Hair

D. Hair Fetishistic disorder is a paraphilic disorder that is diagnosed in patients with recurrent and intense sexual arousal from either nonliving objects or nongenital body parts. The nonliving object or nongenital body part that leads to the sexual arousal is called the fetish. Common fetishes include women's underpants, bras, shoes, feet, toes, and hair. Fetishistic disorder can manifest with fantasies, urges, or behaviors about the fetish. It is more common in men. The diagnostic criteria for fetishistic disorder from the fifth edition of the Diagnostic and Statistical Manual of Mental Disorders require a symptom duration of at least 6 months and the symptoms must cause significant distress. However, most patients who have a fetish do not have clinically significant distress and thus do not meet the diagnostic criteria. There is limited available data regarding the treatment of fetishistic disorder. Pharmacologic treatments that have been attempted include antidepressants and antiandrogen medications. Psychotherapies that may be used include cognitive behavioral therapy and individual psychodynamic therapy.

A 40-year-old woman presents with an ankle wound that has not healed in months. She has had an extensive workup by multiple physicians and has been treated at wound care centers by specialists on several occasions. She continues to have recurrent infections of the wound, and cultures have always grown atypical pathogens. A review of her medical history shows multiple drug allergies and the involvement of multiple medical professionals for issues that are pervasive and atypical. She has undergone multiple medical procedures over the last few years without any pathology being discovered for her symptoms. Which of the following should be included in the treatment plan as a general strategy in treating this patient? A. Discontinue general medical care B. Emphasize that retaliatory actions can be taken C. Ensure that the patient confesses D. Have one clinician oversee patient management

D. Have one clinician oversee patient management Factitious disorder imposed on self is characterized by an individual creating false medical or psychiatric symptoms and then seeking treatment for those factitious diseases or disorders. Individuals may report factitious symptoms or stimulate symptoms even in the absence of external rewards or gain. Individuals typically present with this disorder in their 20s to 30s. Risk factors for developing this disorder include female sex, single marital status, working in health care, past psychiatric history, and family conflict. The etiology of this disorder is unknown, but it has been associated with multiple factors, including early loss via sickness or death, positive experiences related to being sick, trauma, and attention-seeking behaviors. Patients will present with illness or injuries that commonly fall into the following categories: abdominal pain, joint pain, chest pain, coagulopathy, hypoglycemia, infections, nonhealing wounds, vomiting or diarrhea, weakness, or seizures. Psychiatric factitious symptoms most commonly include bereavement, depression, psychosis, and suicidal ideation. Individuals may tamper with medical instruments or specimens, forge medical records, discontinue medical treatment to worsen an existing condition, or fabricate symptoms. Clinicians should suspect factitious disorder in patients using multiple medical facilities, those with multiple drug allergies, and those who have inconsistencies in the history, examination, and laboratory results. Clinical intervention includes having one clinician overseeing the patient management, consulting psychiatry, and informing all clinicians caring for the patient about the diagnosis. Assessing the risk of suicide and monitoring the patient may be necessary to prevent self-injurious behaviors. Discussing the diagnosis with the patient in a nonjudgmental and nonthreatening way is important in taking steps to treat the patient. Psychotherapy is the standard treatment for factitious disorder.

Your patient, who experienced a collision while driving one month ago, is diagnosed with adjustment disorder. Which of the following is most likely reported during history taking? A. Circumscribed, intense periods of fear related to the collision B. Distressing, intrusive thoughts about the collision C. Flashbacks of the collision D. Maladaptive behaviors related to the distress caused by the collision

D. Maladaptive behaviors related to the distress caused by the collision Adjustment disorder is a behavioral disorder characterized by an emotional or behavioral response to an identifiable stressor. The fifth edition of the Diagnostic and Statistical Manual of Mental Disorders also stipulates the behavioral change must occur within 3 months of onset of the stressor and cannot exceed 6 months in duration after the stressor is resolved. Patients should not be diagnosed with adjustment disorder in instances of bereavement. Subtypes of adjustment disorder include with depressed mood, with anxiety, with mixed anxiety and depressed mood, with disturbance of conduct, with mixed disturbance of emotions and conduct, and unspecified. Those diagnosed with an illness, specifically cancer, are more likely to develop adjustment disorder. This diagnosis may be more prevalent in women and is only second to mood disorders in incidence. Patients who engage in self-harm are also commonly diagnosed with adjustment disorder after psychiatric investigation. Risk factors for emotional distress include a history of psychiatric disorders, history of physical or sexual abuse, cognitive impairment, communication barriers, uncontrolled symptoms of cancer, severe comorbid general medical illness, conflicts with caregiver or family, living alone, and financial problems. Somatic symptoms may be the primary concern with multiple, unrelated concerns in different organ systems. Further evaluation reveals underlying mood or conduct disturbances. The essential component in diagnosing adjustment disorder is the presence of a new stressor within 3 months of symptom onset. Adjustment disorder is diagnosed only when the response to the stressor causes marked impairment in the patient's life and does not adhere to societal or cultural norms of response. Maladaptive behaviors related to the distress can include substance use, such as alcohol or illicit drug use, to relieve anxiety or depressive symptoms related to adjustment disorder, and careful evaluation is warranted to rule out substance use disorders. Psychotherapy is the first-line treatment strategy for managing adjustment disorder. Therapy focuses on enabling the patient to remove the stressor from their life, teaching the patient to adapt to stressors that cannot be removed, and altering the response to the stressor with symptom reduction and behavioral modification. Medication therapy may be used as adjunct management for those whose condition does not improve with therapy alone. Antidepressants (e.g., fluoxetine, sertraline, venlafaxine) can be used to combat depression, while benzodiazepines (e.g., lorazepam, diazepam) may alleviate symptoms of anxiety or insomnia.

A 10-year-old boy presents after being referred by his teacher. He is having difficulty concentrating and sitting still while in school. He often interrupts his teacher and has problems completing in-class assignments. His parents report he has the same problems at home, with frequent outbursts and impulsive behaviors. Which of the following appropriate therapeutics works by blocking the reuptake of norepinephrine and dopamine into presynaptic neurons? A. Atomoxetine B. Clonidine C. Guanfacine D. Methylphenidate

D. Methylphenidate Atomoxetine (A) is a selective norepinephrine reuptake inhibitor. Clonidine (B) and guanfacine (C) are alpha-2 adrenergic agonists. Attention-deficit/hyperactivity disorder (ADHD) is a common neuropsychiatric disorder characterized by inattention and impulsivity that causes significant impairment. The diagnosis is made when a patient has a persistent pattern of inattention and hyperactivity or impulsivity, symptoms are present in two or more settings, and symptoms interfere with daily activities and performance. First-line treatment is generally with amphetamines or methylphenidate. Methylphenidate has an onset of action of about 30 to 60 minutes after administration and peaks at 1 to 2 hours. The mechanism of action is through blocking the reuptake of norepinephrine and dopamine into presynaptic neurons. Adverse effects include insomnia, weight loss, loss of appetite, and nervousness. Fatigue and irritability can occur as a rebound effect when the medication begins to wear off.

A 23-year-old woman presents to the clinic reporting sudden-onset sharp chest pain, dyspnea, racing heartbeat, and a choking sensation. The symptoms lasted about 10 minutes. She has a past medical history including major depressive disorder and states she has never experienced these symptoms before. Current medications include bupropion and oral contraceptives. She does not report illicit drug use, alcohol use, or tobacco use. Vital signs include HR 113 bpm, RR 19 breaths per minute, BP 123/91 mm Hg, T 98.8°F, and SpO2 99% on room air. Preliminary ECG readings are shown above, and cardiac enzymes and d-dimer are within normal limits. Which of the following is the most likely diagnosis? A. Factitious disorder B. Illness anxiety disorder C. Myocardial infarction D. Panic attack

D. Panic attack A panic attack is a discrete period of intense fear that is experienced in conjunction with specific cognitive and somatic symptoms. Cognitive symptoms frequently include racing thoughts, preoccupation with health concerns, and catastrophic misinterpretation of somatic symptoms, among others. Patients often report somatic symptoms, such as a choking sensation, racing heartbeat, sweating, nausea, shaking, chest pain, and numbness. Panic attacks are often self-limited and can be categorized as unexpected, situationally bound, or situationally predisposed. Common comorbid anxiety disorders include panic disorder, social phobia, post-traumatic stress disorder, and obsessive-compulsive disorder. Identifying specific triggers for panic attacks can help to uncover these underlying comorbid disorders and further direct specific treatment. Patients who experience recurrent, uncued panic attacks should be treated for panic disorder with primary treatment involving antidepressants, such as fluoxetine, paroxetine, and sertraline. Acute panic attacks can be treated with benzodiazepines (e.g., diazepam, lorazepam), but long-term use of these agents is not recommended due to dependency and side effects (e.g., disinhibition, ataxia, dysarthria, nystagmus). Overdose of benzodiazepines can result in respiratory depression, hypotension, shock syndrome, coma, and death. Factitious disorder (A) is a behavioral disorder in which an individual deceptively presents himself or herself as ill, impaired, or injured even in the absence of an external reward. The patient in the vignette above does not have obvious deceptive behavior and, despite normal preliminary examination and test results, is not overtly falsifying her symptoms. Illness anxiety disorder (B) is characterized by a preoccupation with having or acquiring a serious illness and does not present with discrete episodes of intense fear, such as that demonstrated in the vignette above. Additionally, somatic symptoms are often minimal or even absent in patients with illness anxiety disorder. Myocardial infarction (C) should be thoroughly ruled out in any patient with signs and symptoms suggestive of underlying ischemia. In the vignette above, myocardial infarction is not as likely to be the cause of the patient's symptoms as her preliminary laboratory and ECG results are within normal limits.

A 27-year-old woman presents to the emergency department reporting dysphoria, muscle aches, yawning, diaphoresis, restlessness, rhinorrhea, lacrimation, nausea, vomiting, and diarrhea. Her symptoms began about 12 hours ago. On physical exam, she is tachycardic and hypotensive, with decreased skin turgor, increased bowel sounds, and several scabs and scars noted over both antecubital fossae. Which of the following signs and symptoms is consistent with the patient's most likely diagnosis? A. Altered mental status B. Hyperpyrexia C. Miosis D. Piloerection

D. Piloerection Acute opioid withdrawal is a syndrome caused by abrupt cessation of short-acting opioids, such as heroin. Opioid withdrawal symptoms usually begin within 12 to 24 hours of the last dose of the opioid, peak at day 3, and taper off by day 7. Classic opioid withdrawal signs and symptoms include piloerection, diaphoresis, yawning, restlessness, dysphoria, myalgia, arthralgia, rhinorrhea, lacrimation, nausea, vomiting, diarrhea, palpitations, mydriasis, insomnia, tremor, and irritability. Tachycardia and hypotension can result if the patient becomes dehydrated due to vomiting and diarrhea. Many patients with opioid use disorder continue to use opioids, not for the euphoria, but to avoid the withdrawal symptoms. Treatment of acute opioid withdrawal that is not secondary to opioid antagonist administration is with opioid agonists, such as methadone or buprenorphine, which decrease withdrawal symptoms without causing euphoria. Other nonopioid agents that treat the symptoms of acute opioid withdrawal include promethazine, loperamide, diazepam, and clonidine. Patients who desire continued assistance to become opioid-free can enter methadone programs and receive methadone on a prolonged and gradually diminishing basis, with the eventual goal of total abstinence from opioids.

A 27-year-old woman with a history of sexual assault 4 months ago presents to the clinic with concerns regarding frequent nightmares and anxious mood that began shortly after the assault. She states she is always feeling "on edge" due to frequent flashbacks and has since avoided going anywhere alone, as this triggers her anxiety. Which of the following is the most likely diagnosis? A. Acute stress disorder B. Adjustment disorder C. Panic disorder D. Post-traumatic stress disorder

D. Post-traumatic stress disorder Post-traumatic stress disorder (PTSD) is a disorder that is the result of experiencing or witnessing a traumatic or stressful event, including actual or threatened death, serious injury, or sexual violence. PTSD is more common in women than in men. Symptoms often include intrusive nightmares or flashbacks, avoidance of triggering stimuli, negative alterations in thought process or mood, and increased arousal or hypervigilance. Diagnosis is made using the following criteria: exposure to death, injury, sexual violence, presence of at least one intrusion symptom (distressing dreams, dissociative symptoms), avoidance of stimuli associated with the traumatic event, alterations in arousal and reactivity (exaggerated startle response, hypervigilence). Duration of symptoms must be more than 1 month and cause significant distress and impairment, and must not be better explained by a substance or other medical condition. Treatment includes trauma-focused psychotherapy that can include exposure therapy, cognitive behavioral therapy, or a combination of the two. Eye-movement desensitization and reprocessing may also be helpful. A selective serotonin reuptake inhibitor may be helpful in addition to therapy, or when therapy is not available. Prazosin may be a helpful adjunct medication to treat nightmares and hypervigilance. While benzodiazepines and atypical antipsychotics have been used as adjunct medications previously, they are no longer recommended, as studies have shown that adverse effects outweigh potential benefits. It is important to monitor patients with post-traumatic stress disorder for common comorbid disorders, including major depression and generalized anxiety disorder, and to watch for substance use disorder, as many of these patients self-medicate with substances. Acute stress disorder (A) has similar symptoms of less than 1-month duration. Adjustment disorder (B) is an emotional or behavioral response to a stressor (job loss, divorce, physical illness, etc.) rather than a trauma that causes a response that is disproportionate to the severity of the stressor and usually resolves within 6 months of the stressor. Panic disorder (C) is an anxiety disorder consisting of periods of recurrent and unexpected intense fear and panic that occur without an identifiable trigger and is unrelated to a traumatic event.

A 30-year-old woman presents to the clinic reporting fatigue, excessive sleepiness, decreased appetite, depressed mood, increased guilt, lack of interest in enjoyable activities, and thoughts of suicide. She states these symptoms started 1 week after giving birth to her first child and have persisted for the past 7 weeks. Which of the following represents the most likely diagnosis? A. Peripartum bipolar disorder, manic episode with psychotic features B. Peripartum major depressive disorder with psychotic features C. Postpartum blues D. Postpartum unipolar major depression

D. Postpartum unipolar major depression Postpartum unipolar major depression is characterized by symptoms of major depressive disorder that present themselves during the 12 months after childbirth. The patient must have five or more symptoms that qualify as major depressive symptoms for the diagnosis. These symptoms include 2 or more weeks of depressed mood, excessive guilt, thoughts of suicide or self-harm, alterations in sleep, increased or decreased appetite, anhedonia, psychomotor agitation, fatigue, and diminished concentration. Patients at risk for postpartum depression are those with a history of a depressive episode, those with increased life stressors, patients with little social support, those with gestational diabetes, or patients with multiple gestation. Treatment of postpartum depression includes cognitive behavioral therapy and selective serotonin reuptake inhibitors. Sertraline is considered first-line treatment in nursing mothers with postpartum depression because its excretion into the breastmilk is low. Peripartum bipolar disorder, manic episode with psychotic features (A) presents in the pre- or postpartum period with signs of mania, such as increased energy, expansive mood, irritability, talkativeness, distractibility, and decreased need for sleep. These patients must also exhibit psychotic features, such as auditory or visual hallucinations, delusions, or paranoia. Peripartum major depressive disorder with psychotic features (B) is a diagnosis that may be made when criteria are met for major depressive disorder in the pre- or postpartum period and psychotic features are also present. Postpartum blues (C) refers to depressed mood, irritability, sleep disturbance, fatigue, and decreased concentration that occurs naturally 2 to 3 days after delivery. Postpartum blues are self-limited and mild, disappearing by 2 weeks postpartum.

A 28-year-old man with a history of substance use presents to the emergency department due to headache, nausea, vomiting, and restlessness. He is agitated upon arrival, making it difficult to obtain the history. Physical exam findings include mydriasis, pallor, tremors, and diaphoresis. Vital signs reveal blood pressure of 175/98 mm Hg and pulse of 110 bpm. Urine drug screen is positive for cocaine. Which of the following medications should be avoided in the acute phase of treatment? A. Aspirin B. Bromocriptine C. Nitroglycerin D. Propranolol

D. Propranolol Aspirin (A) is used to treat chest pain that often accompanies cocaine intoxication. Cocaine can cause thrombus formation, making aspirin an important part of the initial intervention. Aortic dissection can also occur due to cocaine use, so if there is any clinical suspicion for aortic dissection, then aspirin should not be used. Aspirin is also used along with statin therapy to prevent secondary cardiovascular disease in the long-term treatment of cocaine use disorder. The euphoria associated with cocaine intoxication is related to the blockage of dopamine reuptake. The dopamine agonist bromocriptine (B) can be used to relieve cravings that occur during acute cocaine withdrawal. Nitroglycerin (C) may be given by either sublingual or intravenous route to individuals with ischemic chest pain and ST elevation or depression. It is safe and typically part of the initial intervention for cocaine toxicity and chest pain when there is a normal ECG or minimal changes in the ST segment.

A 30-year-old woman with a history of anxiety presents to the clinic with postprandial regurgitation that occurs 5 minutes after most meals. She reports no associated retching or dysphagia. High-resolution postprandial impedance manometry shows reflux extending to the proximal esophagus with an increase in gastric pressure. Which of the following is the most likely diagnosis? A. Achalasia B. Esophageal stricture C. Gastroesophageal reflux disease D. Rumination syndrome

D. Rumination syndrome Rumination syndrome is defined as a functional gastrointestinal disorder in which ingested food regurgitates into the mouth after meals. The suspected pathogenesis involves a pressure gradient created by an increase in intra-abdominal pressure (due to abdominal wall activation) and a decrease in intrathoracic pressure. This pressure gradient causes ingested gastric contents to move backward into the mouth. Risk factors for rumination syndrome include anxiety, depression, and constipation. Rumination syndrome classically presents with regurgitation within 10 minutes of finishing most meals. The regurgitation occurs without any effort or retching, and the material can either be spat out or re-swallowed. The diagnosis of rumination syndrome is made clinically. The pediatric and adult diagnostic criteria vary slightly, but they include repetitive regurgitation soon after ingested meals that is not preceded by retching or explained by another medical condition. High-resolution impedance manometry is a diagnostic test that measures pressure and is often performed in patients with regurgitation to rule out achalasia. The classic postprandial impedance manometry finding in rumination syndrome is reflux extending to the proximal esophagus that is associated with an increase in gastric pressure. An upper esophagogastroduodenoscopy is also frequently performed in patients with regurgitation to rule out structural esophagogastric disorders. The management of rumination syndrome consists of treating underlying psychiatric disorders and educating patients to perform diaphragmatic breathing. Diaphragmatic breathing consists of intentionally engaging the abdominal muscles and the diaphragm to fill the lungs more efficiently. This practice helps reduce postprandial intragastric pressure and increases the pressure at the esophagogastric junction. Baclofen can be used in patients with refractory symptoms. Achalasia (A) is a condition marked by impaired relaxation of the lower esophageal sphincter and decreased peristalsis in the distal esophagus due to progressive degeneration of the ganglion cells in the myenteric plexus. Dysphagia is the classic finding, although regurgitation is possible. Furthermore, the classic manometry finding in achalasia is aperistalsis of the distal esophagus and increased lower esophageal sphincter pressure. Esophageal stricture (B) is defined by an abnormal narrowing of the esophageal lumen. It typically presents with dysphagia and can be identified on an upper gastrointestinal series or esophagogastroduodenoscopy. Gastroesophageal reflux disease (C) occurs due to inappropriate relaxation of the lower esophageal sphincter, which results in gastric contents moving backward into the esophagus. Most patients with gastroesophageal reflux disease have retrosternal burning chest pain. Regurgitation is uncommon and is typically a severe finding that may indicate complications, such as a stricture.

A 24-year-old man presents to the psychiatry clinic with his wife for follow-up. He was brought to the emergency department 3 months ago after the police found him blocking traffic on the highway and telling drivers that the apocalypse was coming. At that time, he says he could hear voices telling him to save the world. His wife reports that these behaviors began abruptly 3 months ago after his father died, and he was subsequently treated in a psychiatric hospital, where his symptoms improved. However, he has been withdrawn with a flat affect since then and has not been able to function well enough to find a job. The patient had a negative medical workup near the onset of symptoms, and it included a negative drug test result. He reports no depressed mood. Which of the following is the most likely diagnosis? A. Bipolar disorder type I B. Brief psychotic disorder C. Schizophrenia D. Schizophreniform disorder

D. Schizophreniform disorder Bipolar disorder type I (A) is diagnosed in patients who have had at least one manic episode. However, most patients with bipolar disorder type I have depressive episodes. This patient had psychotic features, such as auditory hallucinations and the delusion that an apocalypse was coming. Psychotic features may be seen during both manic episodes and in patients with schizophrenia who do not meet the criteria for a manic episode. Manic episodes are defined by a persistently elevated mood lasting at least a week and including at least three of the following findings: distractibility, impulsive behavior, grandiosity, flight of ideas, increased activity, insomnia, and rapid or pressured speech. These criteria are not explicitly met in the vignette, thus schizophreniform disorder is the most correct answer. Brief psychotic disorder (B) is a condition in which the symptoms last more than a day but less than a month. The patient must have at least one of the following symptoms: hallucinations, delusions, disorganized speech, and disorganized behavior. Negative symptoms are less likely to occur than in schizophrenia and schizophreniform disorder. Schizophrenia (C) is a psychiatric condition with similar clinical symptoms to schizophreniform disorder. However, the symptoms must persist for at least 6 months to meet the diagnostic criteria.

A 35-year-old man presents with a persistent pattern of odd behavior since early adulthood. He often believes that people talking on the radio have special messages for him. In addition, he has few close friends, pervasive suspicion of others, and social anxiety. Which of the following personality disorders does this individual most likely have? A. Avoidant B. Paranoid C. Schizoid D. Schizotypal

D. Schizotypal Schizotypal personality disorder is a cluster A personality disorder marked by cognitive-perceptual abnormalities, oddness or disorganized behavior, and interpersonal difficulties. The cognitive-perceptual abnormalities include odd beliefs, unusual perceptual experiences, ideas of reference, and paranoia. Ideas of reference are the belief that irrelevant occurrences in the world are related directly to you. Oddness and disorganized behavior manifest predominantly in ways that can be seen, such as hygiene, attire, and social behaviors that are eccentric. Individuals with schizotypal personality typically have few close relationships and chronic social anxiety that does not improve with familiarity. Similar to other personality disorders, the features of schizotypal personality disorder must be present by early adulthood. The diagnosis of schizotypal personality disorder is made according to the criteria in the fifth edition of the Diagnostic and Statistical Manual of Mental Disorders, which are based on the manifestations described above. The recommended treatment of schizotypal personality disorder is psychotherapy with pharmacotherapy targeting specific symptoms. Cognitive-perceptual and psychotic symptoms causing significant distress are typically treated with atypical antipsychotics, such as quetiapine. Patients with prominent cognitive deficits may also be treated with stimulant medications, such as methylphenidate. Guanfacine is an alternative for patients with a history of substance use. Social anxiety in patients with schizotypal personality disorder may be treated with a daily dose of clonazepam. Selective serotonin reuptake inhibitors can be prescribed for patients with a history of substance use disorder. Avoidant (A) personality disorder is marked by social inhibition and fear of rejection. These individuals avoid social interactions and seek jobs with limited interaction with others because of their social anxiety. Despite this anxiety, these individuals desire companionship. The odd behavior and cognitive-perceptual abnormalities distinguish schizotypal personality disorder from avoidant personality disorder. Paranoid (B) personality disorder is marked by pervasive distrust and suspicion of others (without evidence). The odd behavior and cognitive-perceptual disturbance distinguish paranoid personality disorder from avoidant personality disorder. Schizoid (C) personality disorder is also marked by social withdrawal. However, these individuals do not have a desire for companionship.

A 32-year-old woman presents to the emergency department with mental status changes, including agitation and confusion. Her vital signs reveal tachycardia and hyperthermia. On a physical exam, the patient is flushed and dilated pupils are noted. She has hyperreflexia and an inducible clonus. Her medical history includes gastroparesis and generalized anxiety disorder for which she takes metoclopramide and sertraline. Which of the following is the most likely diagnosis? A. Anticholinergic toxicity B. Malignant hyperthermia C. Neuroleptic malignant syndrome D. Serotonin syndrome

D. Serotonin syndrome Anticholinergic toxicity (A) is associated with the use of anticholinergic medications. The similarities between anticholinergic toxicity and serotonin syndrome include hyperthermia, agitation, dry mucous membranes, and mental status changes. Muscular tone and reflexes are normal in anticholinergic toxicity, which differentiates the two syndromes. Malignant hyperthermia (B) presents with muscle rigidity, tachycardia, and hyperthermia after exposure to halogenated anesthetics or depolarizing muscle relaxants. Neuroleptic malignant syndrome (C) develops more gradually than serotonin syndrome, which typically develops over 24 hours. This syndrome is also associated with slow neuromuscular activity rather than neuromuscular hyperactivity.

A woman presents to the clinic 1 hour late for her scheduled appointment. When questioned as to why she is late, she states she had to walk from home because she is afraid of driving. She states the last time she drove, she had a choking sensation, rapid heart rate, sweating, and nausea. Which of the following is the most likely diagnosis? A. Agoraphobia B. Panic disorder C. Social phobia D. Specific phobia

D. Specific phobia Specific phobia is an anxiety disorder in which patients experience intense fear or anxiety about a particular object or circumscribed situation, other than a social situation. These fears commonly involve animals (e.g., spiders, dogs), the natural environment (e.g., storms), blood-injection-injury (e.g., seeing blood, getting an injection), or situations (e.g., enclosed spaces, heights). Young women with low socioeconomic status are at increased risk for developing specific phobias. Phobias tend to appear in middle childhood or early adolescence and persist throughout the patient's lifetime. Impairment related to the specific phobia can range from mild to severe, with many patients initially presenting for an alternative reason, such as a comorbid mood disorder or result of extreme tactics to avoid the phobic trigger. The fifth edition of the Diagnostic and Statistical Manual of Mental Disorders provides criteria for diagnosing specific phobia. These criteria specify that a patient must exhibit a marked fear or anxiety of a specific object or situation, immediate fear and anxiety when confronted with the phobic object or situation, and an avoidance behavior of the phobic situation or object. If the phobic object or situation cannot be avoided, patients express intense fear or anxiety to endure the phobia. The fear or anxiety must be out of proportion to the actual danger posed by the phobic stimuli, present for 6 months or more, and result in impairment or significant distress. First-line treatment for specific phobia is psychotherapy, which should involve exposure to the feared stimulus. If cognitive behavioral therapy is not accessible to the patient or the phobic stimulus is infrequent or rare, benzodiazepines (e.g., diazepam, lorazepam) can be used to treat an anticipated phobic stimuli encounter. For example, a patient can be instructed to take these agents prior to flying if their phobia involves riding in an airplane. Agoraphobia (A) is the fear of being in a situation where help may be difficult or impossible to obtain if panic-like symptoms or other incapacitating or embarrassing symptoms emerge. Patients with this condition often avoid public spaces and crowds. The patient in the vignette has a fear specific to driving motor vehicles and does not express panic or fear in other public situations. Panic disorder (B) presents with similar symptoms to those being described in the vignette, but the panic attacks associated with panic disorder are unprovoked and have no known trigger. In contrast, panic attacks related to specific phobia can be traced to a single triggering object or situation (e.g., driving). Social phobia (C), also known as social anxiety disorder, is defined by excessive fear of scrutiny, embarrassment, and humiliation in social or performance situations. The patient in the vignette is concerned with driving the motorized vehicle.

A 20-year-old man presents to the emergency department complaining of being given some "bad stuff" to smoke at a party. The patient exhibits paranoia, avoidance of eye contact, sedation, diaphoresis, vomiting, bradycardia, and hypotension. He complains of extreme muscle pain and states he knows people are thinking evil thoughts about him. A urine toxicology screen is negative. Which of the following substances is most likely causing his symptoms? A. Bath salts B. Cannabis C. Crack cocaine D. Spice

D. Spice Synthetic cannabinoids are produced in laboratories and come in seven different structures, all of which are dissimilar to naturally occurring marijuana. Their street names include K2, spice, K3, crazy clown, krypton, Aztec fire, and happy tiger incense. Synthetic cannabinoids lack cannabidiol, a flavonoid found in marijuana that has anxiolytic and antipsychotic properties. The synthetic cannabinoids have greater affinity for the CB1 receptors in the brain and can be up to 800 times more potent than natural marijuana. Synthetic cannabinoids are also mixed with varying degrees of herbs that may have their own psychogenic properties. Synthetic cannabinoid intoxication may present with a constellation of symptoms, including bradycardia, tachycardia, hypotension, hypertension, hyperthermia, acute kidney injury, rhabdomyolysis, angina, hypokalemia, hyperglycemia, paranoia, sedation, seizures, altered mental status, diaphoresis, anxiety, psychosis, avoidance of eye contact, nausea, and vomiting. Because synthetic cannabinoids are structurally dissimilar to marijuana (tetrahydrocannabinol), they are not detectable on urine toxicology screens and can only be detected by gas or liquid chromatography-mass spectrophotometry. Treatment of acute intoxication with synthetic cannabinoids is symptomatic and includes intravenous benzodiazepines, as no antidote exists. Bath salts (A) are synthetic cathinones that are also not detectable on urine toxicology screens. The synthetic cathinones are stimulants similar to methamphetamine. Many signs and symptoms of synthetic cathinone intoxication are similar to that of the synthetic cannabinoids, although bradycardia and hypotension are not among them. Synthetic cathinones are not smoked, but rather ingested, insufflated, or administered intramuscularly, rectally, or intravenously. Street names of synthetic cathinones include bath salts, blow, screen cleaner, vanilla sky, bubbles, MCAT, and research chemicals. Cannabis (B) is detectable on urine toxicology screening, as is crack cocaine (C). Crack cocaine is a potent stimulant and is unlikely to cause bradycardia or hypotension.

A 19-year-old woman presents to the clinic complaining of decreased interest in usually enjoyable activities for the past 6 weeks. She also reports that she has gained 10 pounds and has been sleeping 16 hours per day during this time period. She feels fatigued despite sleeping more than usual and has difficulty concentrating when she attempts to be productive. You decide to start her on pharmacologic therapy. Which of the following is a boxed warning of the first-line pharmacologic class used to treat the suspected diagnosis? A. Agranulocytosis B. Diarrhea C. Sexual dysfunction D. Suicidal thoughts

D. Suicidal thoughts Agranulocytosis (A) refers to a blood disorder characterized by a decreased number of granulocytes (neutrophils, eosinophils, and basophils). It is a possible adverse effect of clozapine, which is a second-generation (atypical) antipsychotic. Diarrhea (B) is a possible adverse effect of selective serotonin reuptake inhibitors, particularly sertraline. However, there is not a box warning about diarrhea. Sexual dysfunction (C) is a common adverse effect of selective serotonin reuptake inhibitors, but there is not a box warning about sexual dysfunction. The symptoms of sexual dysfunction may include erectile dysfunction and decreased libido in men and anorgasmia and decreased libido in women.

A 74-year-old man with a history of a psychiatric condition marked by delusions and hallucinations presents to the psychiatric clinic with an insidious onset of involuntary protruding tongue movements. He reports that he has been taking quetiapine for 20 years. Which of the following is the suspected diagnosis? A. Agranulocytosis B. Akathisia C. Neuroleptic malignant syndrome D. Tardive dyskinesia

D. Tardive dyskinesia Schizophrenia is a severe psychiatric condition marked by recurrent episodes of psychosis. Manifestations include hallucinations, delusions, disorganized thoughts, disorganized behavior, and negative symptoms, which are an absence or reduction in normal processes. They include diminished expression, decreased motivation or interest in typically appearing activities, lack of energy, and social isolation. Schizophrenia is diagnosed according to criteria listed in the fifth edition of the Diagnostic and Statistical Manual of Mental Disorders, which include a symptom duration of at least 6 months and the presence of two of the following findings: delusions, hallucinations, disorganized speech, disorganized behavior, and negative symptoms. Patients with schizophrenia are treated with antipsychotic medications, such as quetiapine. Tardive dyskinesia is a medication-induced hyperkinetic movement disorder that is caused by exposure to dopamine-blocking pharmacologic agents. Oral, facial, and lingual dyskinesias (abnormal movements) are the most common manifestations. These movements may manifest as tongue protrusions, smacking the lips, retracting the corners of the mouth, bulging cheeks, or chewing movements. However, patients may also experience dyskinesias in the extremities or trunk or other symptoms, including dystonias (sustained or repetitive muscle contraction), chorea, or tics. The diagnosis of tardive dyskinesia is made clinically. In most cases, the patient will have oral, facial, or lingual dyskinesias and a history of exposure to an antidopaminergic medication, such as an antipsychotic or metoclopramide. Patients who are taking one of these medications should be closely monitored for tardive dyskinesia. The treatment of tardive dyskinesia varies according to the type of presentation. The causative medication should be discontinued if possible, but in many cases, the antipsychotic medication is necessary to treat a psychiatric condition. Patients with minimal symptoms of tardive dyskinesia do not require treatment. Patients with anxiety due to mild symptoms can be treated with a benzodiazepine, such as clonazepam. Those with localized and severe symptoms can be treated with botulinum toxin injections, and patients with disturbing diffuse symptoms can be treated with vesicular monoamine transporter 2 inhibitors, such as valbenazine or tetrabenazine. Anticholinergic drugs, such as benztropine, may exacerbate dyskinesias but can be helpful in patients who primarily have dystonias. Interestingly, some patients who have severe and refractory tardive dyskinesia and no longer require antipsychotic medications are put back on antipsychotic medications because it can help suppress symptoms of tardive dyskinesia. Agranulocytosis (A) refers to a reduced number of granulocyte white blood cells, which includes neutrophils, eosinophils, and basophils. It is a classic side effect associated with the second-generation antipsychotic clozapine. Akathisia (B) is a common extrapyramidal symptom that may occur as an adverse effect of antipsychotic medications. The typical presentation is a motor restlessness described as a compelling urge to move or an inability to sit still. In mild cases, the restlessness is a subjective sensation that cannot be observed. The treatment options for akathisia include beta-blockers, benztropine, and benzodiazepines. Neuroleptic malignant syndrome (C) is a life-threatening neurologic emergency that is associated with antipsychotic medication use. The common manifestations are mental status change (often an agitated delirium), rigidity, fever, and dysautonomia (e.g., tachycardia, hypertension, and tachypnea). Symptoms often occur during the first 2 weeks of therapy with antipsychotics. Treatment consists of stopping the inciting agent and providing supportive care.

Which of the following is the most commonly abused substance by schizophrenic patients? A. Alcohol B. Cannabis C. Cocaine D. Tobacco

D. Tobacco Substance abuse can present with a variety of manifestations, depending on the substance being abused. The most commonly used substances include caffeine, alcohol, and tobacco or nicotine products. The DSM-V provides the diagnosis substance use disorder to broadly classify those displaying addictive behavior. Those at a higher risk of substance abuse include comorbidities of depressive, anxiety, or bipolar disorders, posttraumatic stress disorder, eating disorder, attention deficit hyperactivity disorder, or schizophrenia. In an epidemiologic study, researchers found 90% of schizophrenic patients use nicotine. Screening for substance use disorder should be performed at regular intervals with further assessment as needed. Treatments for tobacco cessation include medications, such as bupropion or varenicline, or nicotine replacement therapy, either alone or in combination with behavioral psychotherapy.

A 14-year-old boy presents to the emergency department with hallucinations after inhaling lighter fluid from a plastic bag. Which of the following is a possible acute toxicity associated with using this type of substance? A. Carbon monoxide poisoning B. Leukoencephalopathy C. Myeloneuropathy D. Ventricular tachydysrhythmias

D. Ventricular tachydysrhythmias Inhalants are volatile substances (gas or chemicals that evaporate at room temperature) that produce vapors that can be inhaled and absorbed through the pulmonary mucosa to produce a transient high. They are typically lipophilic, which leads to their rapid absorption into the brain and a quick high. Children 12-17 years of age and young adults are the most common demographic to use inhalants. Common examples of household products that can be used as inhalants include glues, gasoline, paint thinners, and spray paint. Inhalants can be used in several ways, which include sniffing (inhaled directly from the container), bagging (inhaled from a plastic bag), or huffing (inhaled from a soaked towel or rag). Intoxication with volatile inhalants typically only lasts for a few minutes unless there is continued use. Although most individuals who misuse inhalants do not require medical attention, the health consequences can be life-threatening in some cases. Central nervous system acute manifestations can include euphoria, hallucinations, lethargy, confusion, headache, seizures, and central respiratory depression. The cardiovascular acute manifestations can include ventricular tachydysrhythmias, myocarditis, and sudden cardiac death. Pulmonary manifestations may include pneumonitis and hypoxia. Furthermore, there are several clinical manifestations associated with chronic use. These include leukoencephalopathy, myeloneuropathy, sensorimotor peripheral neuropathy, and hepatotoxicity. The diagnosis of inhalant intoxication or misuse can be difficult to make. Clues that suggest inhalant misuse include chemical odors on the breath, skin, or clothes; empty solvent containers; and the presence of bags, rags, or gauze. The management of acute inhalant intoxication consists of supportive care. Carbon monoxide poisoning (A) can occur acutely after inhalation of methylene chloride, which is metabolized to carbon monoxide. Methylene chloride is an industrial solvent and can be used as a paint thinner. Leukoencephalopathy (B) is a manifestation of chronic inhalant toxicity that occurs after chronic exposure to toluene, which is an inhalant in glues, adhesives, and paint thinners. Leukoencephalopathy is marked by dementia, ataxia, anosmia, and eye movement disorders. Myeloneuropathy (C) is a manifestation of chronic inhalant toxicity that can occur after exposure to nitrous oxide. It is characterized by damage to the tracts of the spinal cord and the peripheral nerves. The manifestations include distal paresthesias, weakness of lower limbs, and an ataxic gait. Which vitamin deficiency causes the neurotoxic effects of nitrous oxide? -Vitamin B12

What are the three clinical findings that comprise the triad associated with Wernicke encephalopathy?

Encephalopathy, oculomotor dysfunction, and gait ataxia

Which antidepressant requires discontinuation for 5 weeks prior to initiating a monoamine oxidase inhibitor?

Fluoxetine

Which antimanic medication has proven to decrease the risk of suicide in patients with bipolar disorder in clinical studies?

Lithium

Which medication for treating bipolar disorder is associated with an increased risk of Ebstein anomaly following in utero exposure?

Lithium

What medication is contraindicated in treating cocaine-induced hypertension?

Propranolol due to unopposed alpha-constriction.

What rare but potentially fatal syndrome may occur during treatment for anorexia nervosa?

Refeeding syndrome

A 28-year-old woman presents to the clinic reporting ongoing intermittent abdominal pain, fatigue, and back pain for the past 7 months. The patient notes she has been evaluated in the emergency department multiple times and has seen other health care clinicians, none of whom have found a cause of her pain. She is constantly worried about her symptoms. On physical exam, the patient appears anxious. Otherwise, there are no significant findings. Labs and imaging results that she had completed recently are all within normal range. Which of the following is the most appropriate initial treatment for this patient? A. Antidepressant therapy B. Cognitive behavior therapy C. Dialectical behavior therapy D. Regularly scheduled visits in the office

Regularly scheduled visits in the office The patient in the above vignette has somatic symptom disorder, which is when an individual has one or more somatic symptoms that are not explained by a physical or medical condition. These symptoms cause significant distress or dysfunction on a day-to-day basis. It is important to suspect this disorder when a patient presents with vague symptoms and an inconsistent history. According to the fifth edition of the Diagnostic and Statistical Manual of Mental Disorders (DSM-5), the diagnostic criteria for somatic symptom disorder must include the following: 1) one or more somatic symptoms that cause the patient distress or psychosocial impairment; 2) excessive thoughts, feelings, or behaviors associated with the somatic symptoms; and 3) although the symptom may change, the disorder must be present for 6 or more months. Initial treatment includes regularly scheduled visits with a health care clinician.

What are the established adverse effects of selective serotonin reuptake inhibitors?

Sexual dysfunction, drowsiness, insomnia, weight gain, headache, and dizziness.

How is factitious disorder distinguished from malingering?

The fabrication of symptoms in malingering is motivated by a clear external reward, such as time off from work or opioid pain medications.

True or false: antidepressants play a role in the treatment of cyclothymic disorder.

True. They may be used judiciously as second- or third-line therapy.

Which teratogenic anticonvulsant has been linked to autism spectrum disorder?

Valproate


Related study sets

Chapter 2: Causes of Abnormal Behavior

View Set

EMSU: Quiz 15, EMT Chapter 15 Respiratory Emergencies

View Set

APHUG Unit 4 - Chapter 8 Political Geography Key Issues #2

View Set

Cognition, Vision, Somatosensory Competency

View Set